You are on page 1of 43

Rights and Duties in Indian

Lesson 3 Constitution
What are Rights? .............................................................. 2 Criticism of Fundamental Rights in Indian
Rights under Indian Constitution ..................................... 2 Constitution ........................................................... 22

Fundamental Rights ............................................. 3 Duties of an Indian Citizen ................................. 23


Overview of Fundamental Rights ............................. 3 List of fundamental duties ..................................... 24

Article 12: Definition of State................................... 5 Features and significance of Fundamental duties . 25

Article 13: Laws inconsistent with or in derogation Criticisms of Fundamental duties .......................... 25
of the Fundamental rights ....................................... 5 Directive Principles of State Policy ..................... 27
Article 14: Equality before law ................................. 5 Constitutional Provisions ....................................... 28
Article 15: Prohibition of Discrimination on Certain Critical evaluation of directive principles .............. 31
Grounds .................................................................... 7 Fundamental Rights vs. Directive Principles .......... 31
Article 16: Equality of Opportunity in matters of Certain directive outside Part IV of the Constitution
Public Employment .................................................. 8 ............................................................................... 32
Article 17: Abolition of Untouchability .................... 9 MCQs for practice .......................................................... 33
Article 18: Abolition of Titles..................................10 Answer Writing Practice ................................................ 36
Article 19: Protection of certain rights regarding Answers & Explanation .................................................. 37
freedom of speech, etc. .........................................10
Article 20: Protection in Respect of Conviction for
Offences .................................................................13
Article 21: Protection of life and personal liberty..13
Article 21A: Right to Education ..............................15
Article 22: Protection against arrest and detention
in certain cases .......................................................15
Article 23: Prohibition of Traffic in Human Beings
and Forced Labour .................................................16
Article 24: Prohibition of employment of children in
factories, etc...........................................................16
Article 25: Freedom of conscience and free
profession, practice and propagation of religion...17
Article 26: Freedom to manage religious affairs ....17
Article 27: Freedom from Taxation for Promotion of
a Religion ................................................................18
Article 28: Freedom from Attending Religious
Instruction ..............................................................18
Article 29: Protection of Interests of Minorities ....18
Article 30: Right of Minorities to Establish and
Administer Educational Institutions.......................18
Article 32: Right to constitutional remedies ..........19
Article 33: Armed forces and fundamental rights .20
Article 34: Restriction on rights when martial law is
in force in any area.................................................20
Article 35: Legislation to give effect to the
provisions of this Part ............................................21
Significance of Fundamental Rights .......................22
1|P a g e W W W . E D U T A P . C O . I N QUERY? HELLO@EDUTAP.CO.IN / 8146207241
▪ According to Heywood:
WHAT ARE RIGHTS?
“Human rights are rights to which people are
A right is described as an entitlement or justified entitled by virtue of being human. They are,
claim to a certain kind of positive and negative therefore, universal rights in the sense that they
treatment from others, to support from others or belong to all human beings rather than to members
non-interference from others. of any particular nation, race, religion, gender, social
▪ Rights are one of the most important conditions class or whatever.”
of social life without which no individual can
generally realize his best self. ▪ They specify the minimum conditions for human
▪ These are the essential conditions for health of dignity and a tolerable life.
both the individual and his society. ▪ Human rights are also fundamental in that they
- When people get and enjoy rights, they can are inalienable, they cannot be taken away.
develop their personalities and contributes ▪ Human Rights are evolved out of self-respect.
their best services to the society. ▪ Rights such as life, liberty and freedom from
▪ Rights are the products of social living and torture are categorised as human rights.
represents claims of the individuals against the
state. CONSTITUTIONAL RIGHTS
▪ Rights can be of various kinds with overlapping All rights that have been conferred to the citizens
relationship. and enshrined in the Constitution of a country are
- A single right can fall into various categories said to be Constitutional Rights.
of rights. ▪ In Indian Constitution, these rights include
- Right to life is a natural right, human right as fundamental rights under Part III as well as
well part of fundamental rights in several other rights mentioned outside Part III.
modern-day States. - So, in India, all fundamental rights are
Constitutional rights but not vice versa.
NATURAL RIGHTS - For example, right to vote or universal adult
They are the basic set of rights which people inherit suffrage is Constitutional right guaranteed in
from the nature. In simple words, Natural rights are: our Constitution under Article 326.

“Rights which persons possess by nature that is LEGAL RIGHTS


without the intervention of the agreement, or in the Legal rights are those rights which are accepted and
absence of political and legal institution.” enforced by the state.
▪ Violation of any legal right is punished by law.
▪ Political theories mentions that an individual ▪ Law courts of the state enforce legal rights.
enters into society with certain basic rights and ▪ These rights can be enforced against individuals
that no Government can deny these rights. and also against the Government. It includes:
▪ The modern idea of natural rights grew out of the - Civil rights, which provide opportunity to
ancient and medieval doctrines of natural law. each person to lead a civilized social life.
▪ Right to equality is one important Natural right. - Political rights, by virtue of which
inhabitants get a share in the political
MORAL RIGHTS process.
Moral Rights based on human sense of goodness - Economic rights, which provide economic
and justice. They are guided by human security to the people.
consciousness.
▪ Moral Rights include rules of good conduct,
courtesy and of moral behaviour. RIGHTS UNDER INDIAN
▪ Moral rights not enforced by the force of law.
CONSTITUTION
HUMAN RIGHTS
Any human being as a part of humanity is entitled to Rights of an individual in Indian Constitution are
have certain rights and the state must take divided into categories:
necessary steps for their protection.

2|P a g e W W W . E D U T A P . C O . I N QUERY? HELLO@EDUTAP.CO.IN / 8146207241


1. Fundamental Rights, which are justiciable in ▪ Also, in India, the leaders of the freedom
nature, are incorporated in Part III of Indian movement had realised the importance of rights
Constitution. and demanded that the British rulers should
2. Directive Principles, which are non- respect rights of the people.
justiciable, are mentioned in separate Part ▪ The Motilal Nehru committee had demanded
IV. a bill of rights as far back as in 1928.
3. Other Constitutional Rights
WHY FUNDAMENTAL?
▪ The fundamental rights and directive principles
The word fundamental suggests that these rights are
follows the preambular assurance of justice,
so important that the Indian Constitution has
equality, liberty, and fraternity.
separately listed them and made special provisions
FUNDAMENTAL RIGHTS for their protection. So, they are fundamental as:
▪ These Rights are provided and protected by the
The fundamental rights are enshrined in Part III of Constitution of the country.
the Constitution from Articles 12 to 35. ▪ They are most essential for the all-round
▪ These rights represent the crystallization of the development of the individuals.
values and concepts held dear in India’s varied ▪ They are the claims of the citizens against the
and rich cultural heritage. State.
▪ They are provided by the Constitution to all ▪ They are part of the basic structure of Indian
persons without any discrimination. Constitution.

While Ordinary rights may be changed by the


Article 14-
•Right to Equality legislature by ordinary process of law making, but a
18
fundamental right may only be changed by
amending the Constitution itself.
Article 19-
•Right to Freedom
22
UTILITY OF FUNDAMENTAL RIGHTS
Fundamental Rights are meant for promoting the
ideal of political democracy.
•Right against exploitation
Article 23- ▪ They uphold the equality of all individuals, the
24
dignity of the individual, the larger public
interest and unity of the nation.
Article 25-
•Right to freedom of Religion ▪ They prevent the establishment of an
28
authoritarian and despotic rule in the country.
▪ They represent the values and aspirations that
Article 29-
•Cultural and Educational Rights should guide all the citizens of India.
30
FEATURES OF FUNDAMENTAL RIGHTS

•Right to Constitutional Remedies COMPREHENSIVE AND DETAILED


Article 32 ▪ The Rights enumerated in the Part III of the
Constitution are very elaborate.
- Each Article has been described with its
▪ The Right to property (Article 31) was deleted
scope and limitations.
from the list of fundamental rights by the 44th
Constitutional Amendment Act, 1978. QUALIFIED, NOT ABSOLUTE
▪ It was made a legal and Constitutional right ▪ Fundamental Rights are not absolute or
under Article 300 (A) in Part XII of the unlimited rights.
Constitution. - They are qualified with limitations and
reasonable restrictions in the collective
OVERVIEW OF FUNDAMENTAL RIGHTS interest of the society.
- The reasonableness of the limitations will be
SOURCE
decided by the Courts.
They are inspired from the Bill of Rights of the
American Constitution. JUSTICIABILITY OF RIGHTS

3|P a g e W W W . E D U T A P . C O . I N QUERY? HELLO@EDUTAP.CO.IN / 8146207241


▪ They are justiciable, allowing persons to move ▪ Protection of language, ▪ Right against
the courts for their enforcement, if and when script, and culture of exploitation
minorities (Articles 23–24)
they are violated.
(Article 29)
- They are defended and guaranteed by the
Supreme Court. ▪ Right of minorities to ▪ Right to freedom of
establish and administer religion
▪ However, when they are violated the aggrieved
educational institutions (Article 25– 28)
person can either move to High Court or (Article 30)
Supreme Court for remedies.
SUSPENSION OF RIGHTS
ENFORCEABILITY OF RIGHTS
▪ They can be suspended during the operation of
Most of the fundamental rights are self-executory in
a National emergency except the rights
nature while a few of them can be enforced on the
guaranteed by Articles 20 and 21.
basis of a law made for giving effect to them.
▪ Article 19 can be suspended only when
▪ Such laws can only be made by the Parliament
emergency is declared on the grounds of war or
not State legislatures.
external aggression.
▪ This is done to ensure uniformity in their
▪ Also, their application can be restricted while
application.
martial law is in force in any area.
FUNDAMENTAL RIGHTS ARE AMENDABLE
NEGATIVE IN CHARACTER
▪ Parliament has the power to amend any part of
▪ They place certain limitations on the authority
the Constitution including fundamental rights.
of the State.
- This can be done without affecting the ‘basic
▪ However few rights confers certain privileges to
structure’ of the Constitution.
the persons.
▪ Parliament can curtail or repeal any of the
fundamental right but only by a Constitutional SPECIAL RIGHTS FOR MINORITIES
Amendment Act and not by an ordinary Act. Constitution guarantees some special rights to the
▪ So, while having very high utility, these rights are minorities of various kinds.
not sacrosanct or immutable. ▪ Cultural and educational rights have been
granted to them.
WIDE SET OF BENEFICIARIES
▪ It abolishes untouchability and makes it a crime.
▪ Some of the fundamental rights in our
▪ It has also granted special protections to
Constitution are available only to the citizens.
women, children, and the weaker sections of
▪ While others are available to all persons whether
society.
citizens, foreigners or legal persons like
corporations or companies. SUBJECTED TO EXCEPTIONS
Available to citizens only Available to citizens and The arena of fundamental rights has been narrowed
foreigners down by subsequent Amendments which added
▪ Prohibition of ▪ Equality before law certain exceptions in the form of Article 31 A, 31 B
discrimination on various and equal protection and 31 C.
grounds of laws
(Article 15) (Article 14)
▪ Equality of opportunity in ▪ Protection in respect Most of them are available against the arbitrary
matters of public of conviction for action of the State, with a few exceptions like those
employment offences against the State’s action and against the action of
(Article 16) (Article 20) private individuals.
▪ Protection of six rights ▪ Protection of life and
regarding freedom of : personal liberty
1. Speech and (Article 21)
expression, ▪ Right to elementary
2. Assembly, education
3. Association, (Article 21A)
4. Movement, ▪ Protection against
5. Residence, and arrest and detention
6. Profession in certain cases
(Article 19) (Article 22)

4|P a g e W W W . E D U T A P . C O . I N QUERY? HELLO@EDUTAP.CO.IN / 8146207241


It is the actions of all above mentioned agencies that
QUESTION
can be challenged in the courts for violating the
Q. A British Citizen staying in India cannot claim
Fundamental Rights.
Right to: (UPSC 1999)
(a) Freedom of trade and profession ARTICLE 13: LAWS INCONSISTENT
(b) Equality before the Law WITH OR IN DEROGATION OF THE
(c) Protection of life and personal liberty FUNDAMENTAL RIGHTS
(d) Freedom of Religion
Article 13 declares that all laws that are inconsistent
Answer: A with or in derogation of the fundamental rights shall
be void.
▪ The power has been conferred on the Supreme
QUESTION Court and the high Courts to declare a law
Q. Which one of the following statements is unconstitutional and void if it is inconsistent with
correct? (UPSC 2017) any of the fundamental rights.
(a) Rights are claims of the State against the ▪ It provides for the doctrine of ‘Judicial review’
citizens. and the definition of ‘law’.
(b) Rights are privileges which are incorporated in
the Constitution of a State.
(c) Rights are claims of the citizens against the JUDICIAL REVIEW
State. Judicial review is the power of the judiciary to examine the
constitutionality of legislative enactments and executive
(d) Rights are privileges of a few citizens against
orders of both the Central and State Governments.
the many. ▪ On examination, if they are found to be violative of the
Constitution, they can be declared as illegal,
Answer: C unconstitutional, and invalid (null and void) by the
judiciary.
ARTICLE 12: DEFINITION OF STATE
▪ The term ‘law’ in Article 13 includes the
In this Part III, the definition of State includes: following:
1. Government and Parliament of India 1. Any Ordinance, Order, Byelaw, Rule,
2. Government and the Legislature of each of the Regulation, Notification, Custom or usage
States having the force of law;
3. All local authorities like District boards, 2. Laws passed or made by a Legislature or
Municipalities and Panchayats other competent authority in the territory of
4. All other authorities within the territory of India India.
or under the control of the Government of India. ▪ It is important to note that a Constitutional
- It also include any authority or body of Amendment is not a law under Article 13 of the
persons which has the power to issue orders, Indian Constitution.
rules and regulations having the force of - So, it cannot be challenged on the grounds
law. that it violates the fundamental rights.
- Even a private body, can be covered under - However, in the Kesavananda Bharati case
‘other authorities’ if it is entrusted with (1973), Supreme Court clarified that:
some public service responsibility as an “A Constitutional Amendment can be challenged on
agency of State. the ground that it violates a fundamental right that
- For example, Life Insurance Corporation of forms a part of the ‘Basic structure’ of the
India, ONGC, Airports Authority comes under Constitution.”
the definition of State. Rights under Article 14-18 are known as “Right to
▪ However, Supreme Court held that a body like Equality”.
the Council of Scientific and Industrial Research,
registered under the Societies Registration Act, ARTICLE 14: EQUALITY BEFORE LAW
was not a ‘state’.

5|P a g e W W W . E D U T A P . C O . I N QUERY? HELLO@EDUTAP.CO.IN / 8146207241


It states that, “the State shall not deny to any person ▪ As this Article refers to ‘person’, so it includes
equality before the law or the equal protection of citizens, foreigners, and other legal persons, like
the laws within the territory of India”. statutory corporations, companies, registered
societies etc.
EQUALITY BEFORE THE LAW
▪ The concept of ‘equality before law’ is an
element of the concept of ‘Rule of Law’ of British QUESTION
origin. As a negative concept it connotes: Q. Which of the following are regarded as the
1. No person is above the law. There is absolute main features of the "Rule of Law"? (UPSC 2018)
supremacy of law as opposed to arbitrary 1. Limitation of Powers
powers. 2. Equality before law
2. Every person is subjected equally to ordinary 3. People's responsibility to the Government
laws of the land. 4. Liberty and civil rights
3. There is no special privileges for anyone. Select the correct answer using the code given
below:
(a) 1 and 3 only
QUICK FACT
The Concept of ‘Rule of Law’ was propounded by A.V. Dicey,
(b) 2 and 4 only
the British jurist. It contains the following three elements: (c) 1, 2 and 4 only
1. Absence of Arbitrary Power. (d) 1, 2, 3 and 4
- Justice must be done through the known principles
of law, as opposed to arbitrariness or wide Answer: C
discretionary powers in the hands of the
Government. EXCEPTIONS TO EQUALITY
2. Equality before the Law. The rule of equality before law has following
3. Primacy of the rights of Individual. Constitutional and other exceptions to it:
- However, in India primacy of the rights flows from
the Constitution. FOR EXECUTIVES
▪ The President of India and Governor of States
▪ It is a negative concept in the sense that it denies enjoy the following immunities (Article 361):
the State to discriminate between individuals, 1. The President or the Governor are not
on arbitrary basis. personally responsible to any court for the
▪ The Supreme Court held that the concept of exercise and performance of their powers,
‘Rule of Law’ is a basic feature of Constitution. while in office.
2. No court can issue arrest warrants for the
EQUAL PROTECTION OF THE LAWS
▪ Equal protection of the laws owes its origin to President as well as Governor, while in
American Constitution. It is a more positive office.
concept which connotes : 3. No criminal proceedings can be started or
1. Right to equality of treatment in equal continued against the President, or the
circumstances, both in the privileges Governor of a State, in any court during his
conferred and liabilities imposed by the term of office.
laws. 4. Civil proceedings against the President or
2. Among equals, the law should be equal and Governor for things done by him in his
equally administered. personal capacity can be done only with a
3. Like should be treated alike without any prior 2 months’ notice.
discrimination. ▪ No MP / MLA can be held responsible in any
▪ Further, because all persons are not, by nature, court for their votes in the legislatures /
attainment, or circumstances are in the same anything said by them in their respective
positions; Article 14 provides that state can treat legislatures.
different persons differently if circumstances GENERAL EXCEPTION
justify such treatment. ▪ No person can be held liable for civil or criminal
▪ This allows categorization of people, provided action for publication in media of a significantly
there is ‘reasonable’ basis of classification. true report of proceeding in legislatures of
Centre or State.
6|P a g e W W W . E D U T A P . C O . I N QUERY? HELLO@EDUTAP.CO.IN / 8146207241
▪ Article 31-C is an exception to Article 14. (b) For the scheduled castes and scheduled
tribes regarding their admission to
FOR OUTSIDERS
educational Institutions including private
▪ The foreign sovereigns (heads), ambassadors
one.
and diplomats enjoy immunity from criminal and
- However, it is not applicable minority
civil proceedings.
Institutions.
▪ The United Nation organisation and its agencies
▪ The Constitution doesn’t define which social
enjoy the diplomatic immunity in India.
categories constitute the ‘socially and
ARTICLE 15: PROHIBITION OF educationally backward classes’.
DISCRIMINATION ON CERTAIN GOVERNMENT RESPONSE
GROUNDS ▪ Centre enacted the Central Educational
Institutions (Reservation in Admission) Act,
Article 15 states that:
2006, providing a quota of 27% for candidates
1. 15 (1): The State shall not discriminate against
belonging to the Other Backward Classes (OBCs)
any citizen on grounds only of religion, race,
in all Central higher educational institutions.
caste, sex, and place of birth or any of them.
2. 15 (2): No citizen shall, on grounds only of JUDICIAL RULING
religion, race, caste, sex, place of birth or any of In 2008, the Supreme Court directed the Central
them, be subject to any disability, liability, Government to exclude the ‘creamy layer’
restriction, or condition with regard to; (advanced sections) among the OBCs while
(a) access to shops, public restaurants, hotels, implementing the law.
and places of public entertainment;
103 RD AMENDMENT ACT OF 2019
(b) the use of wells, tanks, bathing ghats, roads
This Act provides for:
and places of public resort maintained
▪ Article 15 (6) to provide reservations to
wholly or partly out of State funds or
economically weaker sections (EWS) for
dedicated to the use of the general public.
admission to educational institutions including
IMPORTANT ANALYSIS private educational institutions, whether aided
▪ The use of word ‘only’ is significant in Article 15. or unaided by the State, other than the minority
- It implies that the discrimination on any educational institutions.
other ground is not prohibited. - The amendment aims to provide reservation
▪ While Article 15 uses the word ‘discrimination’, to those who do not fall in 15 (5) and 15(4)
it did not defined it. In simple words it connotes (effectively, SCs, STs and OBCs).
‘to make an adverse distinction.’ ▪ This 10% reservation is in addition to the existing
reservations.
EXCEPTION TO RULE OF NON-DISCRIMINATION
These exceptions are mentioned in the Article 15
itself in provisions 15 (3) and 15 (4). ELIGIBILITY CRITERIA FOR EWS
▪ State can make any special provision for women 1. Persons whose family has gross (income from all
and children. sources) annual income is below Rs. 8 lakh.
2. Persons whose family is having any one of the following
- For example, reservation of seats for women
set of assets (all property held by family is clubbed) is
in Panchayats. included:
▪ State can make special provision for the - Less than 5 acres of agricultural land.
advancement of any socially and educationally - Less than 1000 sq. ft of residential flat.
backward classes of citizens or for the scheduled - Less than 100 sq. yards of residential plot in
notified municipalities.
castes and scheduled tribes.
- Less than 200 sq. yards of residential plot in areas
93RD AMENDMENT ACT OF 2005 other than the notified municipalities.
▪ 15 (5): The State can make special provision for
the:
(a) Advancement of any socially and
educationally backward classes of citizens,
or

7|P a g e W W W . E D U T A P . C O . I N QUERY? HELLO@EDUTAP.CO.IN / 8146207241


27% of Government jobs should be reserved for
QUESTION
them.
Q. Prohibition of discrimination on grounds of
religion etc. (Article 15 of the Constitution of
India) is a Fundamental Right classifiable under RESERVATION
(UPSC 1995) The objective of reservation as envisioned by the founding
(a) The Right to freedom of Religion fathers of the Constitution was to ensure social justice by
giving special status to backward castes as they were
(b) The Right against Exploitation denied equal opportunities for generations and required
(c) The Cultural and Educational Rights special assistance to catch up with the other forward castes.
(d) The Right to Equality
GOVERNMENT’S RESPONSE
Answer: D 1. In 1990 the V P Singh Government declared
reservation of 27% Government jobs for OBCs.
ARTICLE 16: EQUALITY OF - After this, the total reservation for all (OBCs,
OPPORTUNITY IN MATTERS OF PUBLIC SCs and STs) increased to 50%.
EMPLOYMENT 2. In 1991, P. V. Narasimha Rao Government
brought few changes in reservation pattern:
As per the Article 16: - Within the 27% quota to OBCs, preference
1. 16 (1): There shall be equality of opportunity for would be given to the poorer sections of
all citizens in matters relating to employment or OBCs for reservation.
appointment to any office under the State. - Additional 10% reservation quota is
2. 16 (2): No citizen shall, on grounds only of introduced for economically backward
religion, race, caste, sex, descent, place of birth, sections of uncovered castes.
residence, or any of them, be ineligible for, or
discriminated against in respect or, any JUDICIAL SCRUTINY

employment or office under the State. In the Indra Sawhney Case of 1992 (also known as
Mandal Case), the Supreme Court uphold this 27%
EXCEPTIONS TO THIS ARTICLE quota for backward classes with certain conditions
These exceptions are mentioned in the Article 16 like:
itself in provisions 16 (3), 16 (4) and 16 (5). ▪ The advanced sections among the OBCs (the
▪ Residence can be allowed as a condition for creamy layer) should be excluded from the list of
employment for public offices of States/UT/local beneficiaries of reservation.
bodies by an act of Parliament. ▪ No reservation in promotions (as of now).
▪ Reservations of appointments or posts can be ▪ The total reserved quota should not exceed 50%
made for backward classes not adequately except in some extraordinary situations.
represented in State services. ▪ The ‘carry forward rule’ is held valid in case of
▪ Parliamentary law can allow members of a backlog vacancies subject to the limit of 50%.
religious body to be of that religion only. ▪ No reservation for economically backward
MANDAL COMMISSION
sections of uncovered castes.
The President appointed a Backward Class GOVERNMENT RESPONSE
Commission in December 1978 under the 1. Ram Nandan Committee was appointed to
chairmanship of B. P. Mandal. identify the creamy layer among the OBCs. It
▪ It had to determine the criteria for defining submitted its report in 1993, which was
India’s “socially and educationally backward accepted.
classes” and to recommend steps to be taken for 2. National Commission for Backward Classes was
the advancement of those classes. established in 1993 by an Act of Parliament.
▪ Commission through collection of the data & 3. 77th Constitutional Amendment Act, inserted
evidence identified castes that are socially, Clause 4A in Article 16, which enables the state
educationally, and economically backward. to make any law regarding reservation in
▪ The Commission concluded that India’s promotion for SCs and STs.
population consisted of approximately 52 - However, the SC in 1990s restored the
percent OBCs, excluding SCs and STs, therefore seniority of General candidates once
promoted at par with the SC/ST candidates
8|P a g e W W W . E D U T A P . C O . I N QUERY? HELLO@EDUTAP.CO.IN / 8146207241
who got quick promotions ahead of their ARTICLE 17: ABOLITION OF
batch mates. UNTOUCHABILITY
4. However, 85th Constitutional Amendment Act,
2001 gave back “consequential seniority” to As per the Article 17:
SC/ST promotees. 1. Untouchability is abolished and its practice in
any form is forbidden, and
CONSEQUENTIAL SENIORITY 2. The enforcement of any disability arising out of
Consequential seniority allows reserved category untouchability shall be an offence punishable in
candidates to retain seniority over general category peers.
▪ If a reserved category candidate is promoted before a
accordance with law.
general category candidate because of reservation in
promotion, then for subsequent promotion the ▪ The objective of this Article was to end the
reserved candidate retains seniority. inhuman practice of treating certain fellow
▪ For example: beings as untouchable by reason of their birth in
- Consider that Person X, belonging to the General
Category, currently holds Level 1 of a Government
certain caste.
post. ▪ The Supreme Court has held that this right is
- Another person Y, appointed under the Scheduled available against private individuals and it is the
Caste quota, is junior to X in Level 1. Constitutional duty of the State to take
- When promotions to Level 2 are to be decided, let necessary steps to see that this right is not
us assume further, that due to reservations in
promotions Y has to be promoted to Level 2 before
violated.
X because there are no Scheduled Caste candidates
at a seniority similar to that of X. The Untouchability (Offences) Act, 1955 later
- The question that then arose was whether X would modified to read as the Protection of Civil Rights Act
regain seniority over Y when he is promoted to (1955), declares the following acts as offences:
Level 2 in due course.
1. Preventing any person from entering any place
- ‘Consequential Seniority’ means that X will not
regain her seniority and Y will now be considered
of public worship or from worshipping therein;
senior to X within Level 2.” 2. Justifying untouchability on any grounds;
3. Denying access to any shop, hotel or places of
M. NAGARAJ VS. UNION OF INDIA CASE 2006 public entertainment and refusing to sell goods
In this case, Supreme Court has upheld the state’s or render services to any person;
decision to extend reservation in promotion for SCs 4. Insulting a person belonging to scheduled caste
and STs. on the ground of untouchability;
▪ However, the court has also asked the State to 5. Refusing to admit persons in hospitals,
provide data on following three parameters: educational institutions or hostels established
1. Data on the Backwardness of the class for public benefit;
enjoying the benefit of reservation. 6. Preaching untouchability directly or indirectly.
2. Data of Inadequate representation of
SCs/STs in the position/service for which ▪ The term ‘Untouchability’ has not been defined
reservation in promotion is to be granted. either in the Constitution or in the Act.
3. How such reservations in promotions would - It is assumed that the word has a well-known
affect/further administrative efficiency? connotation that refers to the social
▪ However, in 2019, in a ruling, Supreme Court disabilities imposed on certain classes of
turned down the requirement of data on first persons by reason of their birth in
parameter. certain caste.
103 RD AMENDMENT ACT OF 2019 ▪ It is important to note that if a person is
This Act provides for: convicted of the offence of ‘untouchability’, he
▪ Article 16 (6) to provide reservations to shall be disqualified for election to the Union or
economically weaker sections (EWS) in a State legislature.
Government jobs.
- The Amendment aims to provide reservation
to those who do not fall in other categories
(effectively, SCs, STs and OBCs).

9|P a g e W W W . E D U T A P . C O . I N QUERY? HELLO@EDUTAP.CO.IN / 8146207241


▪ Supreme Court in 1996 upheld the
QUESTION
Constitutional validity of the National Awards—
Q. Match List I (Articles of the Constitution of
Bharat Ratna, Padma Vibhushan, Padma
India) with List II (Provision) and select the
Bhushan and Padma Shri.
correct answer using the codes given below the
- They are not violative of Article 18 as the
lists (UPSC 2004)
theory of equality does not mandate that
List I (Articles of List II (Provisions)
Indian Constitution) merit should not be recognised.
A. Article 14 1. The State shall not
discriminate against any citizen
on ground only on religion, race, NATIONAL AWARDS
caste, sex place of birth or any of In 1954, the Government of India introduced National
term. awards or decorations (in the form of medals) of four
B. Article 15 2. The State shall not deny to any categories, namely, Bharat Ratna, Padma Vibhushan,
person equality before the law or Padma Bhushan and Padma Shri.
the equal protection of laws ▪ Bharat Ratna was to be awarded for ‘exceptional
within the territory of India. services towards the advancement of Art, Literature
and Science’, and in recognition of public services of the
C. Article 16 3. Untouchability is abolished higher order.
and its practice in any form is - The recommendations for Bharat Ratna are made
forbidden. by the Prime Minister himself to the President. No
D. Article 17 4. There shall be equality of formal recommendations are necessary.
opportunity for all citizens in ▪ Other three are awarded for “distinguished public
matters relating to employment service in any field, including services rendered by the
or appointment to any office Government servant”, in order of the degree of the
under the state. merit of their service.
- The nomination process is open to the public. Even
Codes: A B C D self-nomination can be made.
(a) 2 4 1 3
(b) 3 1 4 2 ▪ This Act does not bars Academic Institutions to
(c) 2 1 4 3 confer titles or honours by awarding the men of
(d) 3 4 1 2 merit.
▪ Similarly, the State is not debarred from
Answer: C awarding military distinctions like Major etc.
ARTICLE 18: ABOLITION OF TITLES
QUESTION
‘Title’ is something that attached with one’s name as Q. In the Indian Constitution, the Right to
an appendage, like Rai bahadur or Maharaja. Equality is granted by five Articles. They are
▪ This Article abolishes ‘Titles’ and makes (UPSC 2002)
following provisions: (a) Article 16 to Article 20
1. 18 (1): No title, not being an academic or (b) Article 15 to Article 19
military distinction, shall be conferred by (c) Article 14 to Article 18
State. (d) Article 13 to Article 17
2. 18 (2): No citizen of India shall accept any title
from any foreign state. Answer: C
3. 18 (3): A foreigner holding any office of profit
or trust under the State cannot accept any The Rights under Article 19-22 are classified as
title from any foreign State without the “Right to Freedom”.
consent of the President.
4. 18 (4): No person holding any office of profit ARTICLE 19: PROTECTION OF CERTAIN
or trust under the State shall, without the RIGHTS REGARDING FREEDOM OF
consent of the President, accept any present, SPEECH, ETC.
emolument, or office of any kind from or
under any foreign State. Article 19 guarantees to all citizens the six essential
rights. These are
IMPORTANT POINTS 1. Right to freedom of speech and expression.
10 | P a g e W W W . E D U T A P . C O . I N QUERY? HELLO@EDUTAP.CO.IN / 8146207241
2. Right to assemble peaceably and without
arms. This right includes the following:
3. Right to form associations or unions or co-
operative societies. •Right to propagate one’s views as well as views of
4. Right to move freely throughout the territory others.
of India. •Freedom of the press.
5. Right to reside and settle in any part of the •Freedom of commercial advertisements.
territory of India. •Right against tapping of telephonic conversation.
6. Right to practice any profession or to carry on •Right to telecast
any occupation, trade, or business. •Right against bundh called by any organisation.
▪ Earlier, it also contains the ‘Right to property’, •Right to know about government activities.
but later it was deleted by the 44th Constitutional •Freedom of silence.
Amendment Act of 1978. •Right against imposition of pre-censorship on a
▪ These six freedoms are recognised as the newspaper.
‘natural rights inherent in the status of a •Right to demonstration or picketing but not right
citizen’. to strike.
▪ The State can impose ‘reasonable’ restrictions
on the enjoyment of these six rights only on the Grounds of Reasonable Restrictions on
grounds mentioned in the Article 19 itself and this Right:
not on any other grounds. • Sovereignty and integrity of India
- The restrictions should not be arbitrary or • Security of the state
excessive in nature. • Friendly relations with foreign states
- The responsibility of proving reasonableness • Public order
to the satisfaction of Courts lies with the • Decency or morality
State.
• Contempt of court
▪ These six rights are protected against only the
• Defamation
state actions and not private individuals.
• Incitement to an offence.
▪ These rights are available only to the citizens not
foreigners. Also, shareholders, who are citizens
also have these six rights. ▪ This right is a sine qua non of the functioning of
democratic polity as it offers scope for debate
RIGHT TO FREEDOM OF SPEECH AND and discussion.
EXPRESSION ▪ By providing the reasonable restrictions a
It implies that every citizen has the right to express balance should be maintained between freedom
his views, opinions, belief, and convictions freely by of speech and expression and public decency
word of mouth, writing, printing, picturing or in any and morality.
other manner. ▪ Flying of National flag, casting of vote (voter
expresses its choice by casting vote), right to
information (voter’s right to know assets of a
candidate so as to cast his/her choice) are
integral to freedom of expression.

RIGHT TO FREEDOM OF ASSEMBLY


The Constitution guarantees right to hold meetings
and take out processions. The processions and
meetings should be unarmed and peaceful.
▪ An assembly declared unlawful can be validly
banned and can be dispersed.

11 | P a g e W W W . E D U T A P . C O . I N QUERY? HELLO@EDUTAP.CO.IN / 8146207241


RIGHT TO FREEDOM OF MOVEMENT
Grounds of Reasonable Restrictions on Every Citizen can move from one state to another
this Right: and anywhere within a state.
• Sovereignty and integrity of India ▪ He/she is free to move within the country’s
• Public order including the maintenance of traffic in territories without restrictions (Internal
the area concerned. Freedom).
▪ Under Section 144 of Criminal Procedure Code
(1973), a Magistrate can restrain an assembly, Grounds of Reasonable Restrictions on
meeting, or procession if: this Right:
1. There is a risk of obstruction, annoyance or • Interests of general public
danger to human life, health, or safety. • Protection of interests of any scheduled tribe.
2. A disturbance of the public tranquillity or a
riot or any affray. ▪ The Supreme Court held that the freedom of
▪ Under Section 141 of the Indian Penal Code, the movement of prostitutes can be restricted on
assembly of five or more persons can be declared the ground of public health and in the interest of
unlawful if it is aimed: public morals.
1. To resist the execution of any law or legal ▪ Restrictions to protect the interests of the
process. Scheduled tribes have been provided keeping in
2. To forcibly occupy the property of some view mostly the aboriginal tribes which have
person. their own distinct culture, language, and
3. To commit any mischief or criminal trespass. customs.
4. To force some person to do an illegal act. RIGHT TO FREEDOM OF RESIDENCE
5. To threaten the government or its officials on Every citizen has the right to reside and settle in any
exercising lawful powers. part of the territory of the country.
RIGHT TO FREEDOM OF ASSOCIATION
The Constitution declares that all citizens will have This right includes the following:
the right to form Associations or Unions or Co-
operative societies. • Right to reside in any part of the country. It
includes temporary stay.
• Right to settle in any part of the country on
This right includes the following: permanent basis.

• Right to continue with the association or union. Grounds of Reasonable Restrictions on


• Right of not to form or join an association or this Right:
union.
• Interests of general public.
• Protection of interests of any schedule tribe.
Grounds of Reasonable Restrictions on
this Right:
▪ The Supreme Court held that certain areas can
• Sovereignty and integrity of India,
be banned for certain kinds of persons like
• Public order
prostitutes and habitual offenders.
• Morality.
▪ Restrictions on movements of person afflicted
by AIDS have been upheld by Bombay HC.
▪ The Supreme Court held that the trade unions
have no right to strike or right to declare a lock- RIGHT TO FREEDOM OF PROFESSION
out. All citizens are given the right to practise any
▪ Right to obtain recognition of the association is profession or to carry on any occupation, trade, or
not a fundamental right. business.
▪ The Associations under this provisions include ▪ The right to carry on a business includes the right
political parties, societies, clubs, companies, to close it any time the owner likes.
organisations, partnership firms, trade unions - Thus, no citizen can be compelled to carry on
and other similar body of persons. business against his will.
12 | P a g e W W W . E D U T A P . C O . I N QUERY? HELLO@EDUTAP.CO.IN / 8146207241
▪ The only ground of reasonable restrictions on it (b) Subjected to a penalty greater than that
is the interests of general public. which might have been inflicted under the
- This freedom is not applicable for trade or law in force at the time of the act.
business in noxious, hazardous, immoral, or However, this limitation is imposed only on criminal
dangerous goods like; laws and not on civil laws or tax laws.
a. Intoxicating drugs or liquors ▪ Thus, the legislatures are prohibited to make
b. Adulterated foods criminal laws with retrospective effect.
c. Indulging in trafficking in women or children. ▪ This immunity cannot be claimed against
▪ The State is also empowered to: preventive detention or demanding security
1. Prescribe professional or technical from a person.
qualifications necessary for practising any ▪ Similarly, the immunity is granted only in cases
profession or carrying on any occupation, of conviction for criminal offences, not the
trade, or business. trials.
2. Carry on by itself any trade, business,
NO DOUBLE JEOPARDY
industry whether to the exclusion of citizens
▪ No person shall be prosecuted and punished for
or otherwise.
the same offence more than once.
▪ So, the State is competent to nationalise any
It is available only in proceedings before a court of
business wholly or partially by excluding it for all
law or a judicial tribunal not departmental ones.
citizens.
- Thus, a Government servant prosecuted and
- The state is not required to justify its trade
convicted by a court of law can nonetheless
monopoly.
be punished again under departmental
proceedings for the same offence and vice
MAINS QUESTION versa.
Q. What do you understand by the concept
NO SELF-INCRIMINATION
“freedom of speech and expression”? Does it
Under common criminal laws, a person (accused of
cover hate speech also? Why do the films in India
an offence) is presumed to be innocent and need
stand on a slightly different plane from other
not make any statement against his will.
forms of expression? Discuss.
▪ It is the responsibility of prosecution to prove his
(UPSC Mains 2014)
guilt. These principles are embodied in Article 20
(3).
MAINS QUESTION ▪ Accordingly, accused person shall not be
Q. Examine the scope of Fundamental Rights in compelled to be a witness against himself.
the light of the latest judgement of the Supreme 1. It extends only to criminal proceedings and not
Court on Right to Privacy. to civil proceedings.
(UPSC Mains 2017) 2. It does not extend to:
(a) Compulsory production of material objects
(b) Compulsion to give thumb impression,
specimen signature, blood specimens, and
ARTICLE 20: PROTECTION IN RESPECT (c) Compulsory exhibition of the body.
OF CONVICTION FOR OFFENCES - Compulsion for production of documents is
prohibited only if the documents conveys
This Article has taken care to safeguard the rights of the personal knowledge of the accused.
persons accused of crimes. It contains three - Similarly, search of premises of accused
provisions: under a search warrant and seizure of
NO EX-POST-FACTO LAW documents are not violative of Article 20 (3).
▪ No person shall be: ▪ This Article is not violated when a person
(a) Convicted of any offence except for violation volunteers evidence against himself.
of a law in force at the time of the
commission of the act charged as an offence. ARTICLE 21: PROTECTION OF LIFE AND
PERSONAL LIBERTY

13 | P a g e W W W . E D U T A P . C O . I N QUERY? HELLO@EDUTAP.CO.IN / 8146207241


Article 21 states that no person shall be deprived of - Whether the enforcement of law followed
his life or personal liberty except according to the procedure prescribed under the law.
procedure established by law. ▪ If any of the test fails, the court will extend
▪ This right is available to citizens as well as protection to the individual against the arbitrary
foreigners. action of the executive.
▪ However, if all the above 3 tests are found to be
EVOLUTION OF THIS RIGHT
valid, the court will not apply a fourth test
▪ In the Gopalan case (1950), the Supreme Court
whether the law in question is fair, just &
held that:
reasonable.
1. The protection under Article 21 is available
only against arbitrary executive action and DUE PROCESS OF LAW
not from arbitrary legislative action. This doctrine is followed in American Constitution.
2. The ‘personal liberty’ means only liberty In simple terms it can be expressed as:
relating to the person or body of the ▪ Due process of law = Procedure established by
individual. law + law must be fair and just
This is because of the expression ‘procedure ▪ Supreme Court through its various judgements
established by law’ in Article 21. It does not has introduced it in Article 21.
questions the validity of law which has prescribed
the procedure. Judiciary has given the expression ‘Personal Liberty’
▪ In the Menaka case (1978), the Supreme Court in Article 21 the widest amplitude and it covers a
overruled its earlier judgement in the Gopalan variety of rights that go to constitute the personal
case and held that: liberties of a man like:
1. Procedure prescribed by any law should be 1. Right to livelihood
reasonable, fair and just. 2. Right to privacy
2. The protection under Article 21 is available 3. Right to shelter
not only against arbitrary executive action 4. Right to free legal aid
but also against arbitrary legislative action. 5. Right to access to Internet
3. ‘Right to life’ also includes within its ambit 6. Right to travel abroad
the right to live with human dignity and all 7. Right to free trial
those aspects of life which go to make a
man’s life meaningful, complete, and worth
MAINS QUESTION
living.
Q. Examine the scope of Fundamental Rights in
PROCEDURE ESTABLISHED BY LAW the light of the latest judgement of the Supreme
It means if Parliament (legislative body) pass a law, Court on Right to Privacy.
then the life or personal liberty of a person can be (UPSC Mains 2017)
taken off according to the provisions and
procedures of the that law.
FLAW
It does not seek whether the laws made by
Parliament is fair, just, and not arbitrary.
▪ “Procedure established by law” means a law
duly enacted is valid even if it is contrary to
principles of justice and equity.
▪ If the person’s deprivation of life and liberty is
challenged before a court of law, the Court will
apply the following 3 tests:
- Whether there exists any law that deprives
an individual of his life or liberty.
- Whether the legislature that enacted the said
law had the competence to enact such a law.

14 | P a g e W W W . E D U T A P . C O . I N QUERY? HELLO@EDUTAP.CO.IN / 8146207241


1. The right to be informed of the grounds of
QUESTION
arrest.
Q. Right to Privacy is protected as an intrinsic part
2. The right to consult a lawyer of one’s own
of Right to Life and Personal Liberty. Which of the
choice.
following in the Constitution of India correctly
3. To be produced before a nearest magistrate
and appropriately imply the above statement?
within 24 hours of arrest (excluding the holidays
(UPSC 2018)
and time taken during the journey).
(a) Article 14 and the provisions under the 42nd
4. The period of detention cannot be beyond what
Amendment to the Constitution.
is authorized by the magistrate.
(b) Article 17 and the Directive Principles of State
Policy in Part IV. EXCEPTIONS TO ABOVE RIGHTS
(c) Article 21 and the freedoms guaranteed in Part ▪ They apply only to an act of a criminal or quasi-
III. criminal nature.
(d) Article 24 and the provisions under the 44th ▪ These safeguards are not available to an enemy
Amendment to the Constitution. alien or a person arrested or detained under a
preventive detention law.
Answer: C
PROVISIONS RELATED TO PREVENTIVE DETENTION
Preventive detention means detention of a person
ARTICLE 21A: RIGHT TO EDUCATION without trial and conviction by a court.
▪ It is for preventing the occurrence of an illegal
This provision was added by the 86th Constitutional
act and not for punishing a person for any illegal
Amendment Act of 2002. It made elementary
act.
education a Fundamental Right.
▪ This protection is available to both citizens as
well as foreigners.
QUICK FACT ▪ Such a detention is subject to the following
▪ The original Constitution contained a provision for free safeguards:
and compulsory education for children under Article 45 1. A person detained on the ground of suspicion
in Part IV. shall be detained for a maximum period of
▪ However, series of judicial decisions culminated in
three months.
converting a non-enforceable right to education in
Directive Principles of State Policy into an enforceable 2. If the Government seeks to detain the
Fundamental Right, leading to the incorporation of arrested person beyond three months, his
Article 21-A. detention must be authorized by an Advisory
body, comprising judges of the High Court.
▪ As per this Article it is the duty of State to 3. The detained person must be informed
provide free and compulsory education to all about the reason of his arrest, as soon as
children of 6 to 14 years in such manner as the possible.
State, may by law determine. 4. The detained person must have the earliest
▪ In pursuance of Article 21A, the Parliament opportunity to present his case before the
enacted the Right of Children to Free and authority of law.
Compulsory Education (RTE) Act, 2009.
EXCEPTION TO ABOVE RIGHTS

ARTICLE 22: PROTECTION AGAINST ▪ The facts considered to be against the public
ARREST AND DETENTION IN CERTAIN interest need not to be disclosed to the arrested
CASES person.
LEGISLATIVE POWERS FOR PREVENTIVE DETENTION
Article 22 grants protection to persons who are
Legislative powers with regard to Preventive
arrested or detained.
detention has been divided between the Parliament
PROVISIONS RELATED TO ORDINARY LAW and the State legislatures by the Indian Constitution.
The first part of Article 22 provides specific rights to
persons who is arrested or detained under an
ordinary law (punitive detention). These rights are:

15 | P a g e W W W . E D U T A P . C O . I N QUERY? HELLO@EDUTAP.CO.IN / 8146207241


1. Immoral traffic in women and children, including
•Defence prostitution
2. Devadasis – mainly practised in south India.
Parliament •Foreign Affairs
3. Slavery
•Security of India
4. Selling and buying of men, women, and children

FORCED LABOUR
• Security of a state It means compelling a person to work against his will. It can
Both be physical, social, and economic compulsion.
Parliament • Public order
• Maintenance of supplies EXCEPTION
and State and services essential to
Legislature ▪ It permits the State to impose compulsory
the community.
service for public purposes (like military service
or social service) for which it is not bound to pay.
▪ However, the State is not permitted to make any
Also, the Parliament may by law prescribe:
discrimination on grounds only of religion, race,
1. The maximum period for which any person may
caste, or class.
in any class or classes of cases be detained under
any law providing for preventive detention. IMPORTANT POINTS
2. Procedure to be followed by an advisory board ▪ Courts have held that even if some
in an inquiry. remuneration (but below the actual) is paid, the
3. Circumstances and the classes of cases in which labour may still be forced labour.
a person can be detained for more than three ▪ The whole idea is not to allow the state or
months without obtaining the opinion of an anyone to compel a person to work against his
advisory board. will.

Article 23-24 are classified as ‘Rights against ARTICLE 24: PROHIBITION OF


Exploitation’. EMPLOYMENT OF CHILDREN IN
FACTORIES, ETC.
ARTICLE 23: PROHIBITION OF TRAFFIC
IN HUMAN BEINGS AND FORCED It states that no child below the age of fourteen
LABOUR years shall be employed to work in any factory or
mine or engaged in any other hazardous
Article 23 says that: employment.
1. Traffic in human beings, begar and other similar ▪ Parliament has thus enacted Child Labour
forms of forced labour are prohibited. (Prohibition and Regulation) Act, 1986.
2. Any contravention of this provision shall be an ▪ It has been amended in 2016 and renamed the
offence punishable in accordance with law. Principal Act as the Child and Adolescent Labour
▪ These practices militate against human dignity (Prohibition and Regulation) Act, 1986.
enshrined in the preamble. This right is available
to both citizens and non-citizens.
QUESTION
▪ It protects the individual not only against the
Q. Which of the following are envisaged by the
State but also against private persons.
Right against Exploitation in the Constitution of
▪ To punish these acts, the Parliament has made
India?
the Immoral Traffic (Prevention) Act, 1956.
(UPSC 2017)
1. Prohibition of traffic in human beings and
IMPORTANT TERMS forced labour
2. Abolition of untouchability
BEGAR
It means compulsory work without remuneration.
3. Protection of the interests of minorities
▪ Under the old zamindari system, the tenants were 4. Prohibition of employment of children in
sometimes forced to render free service to their factories and mines
landlords. Select the correct answer using the code given
TRAFFIC IN HUMAN BEINGS
below:
It includes: (a)1, 2 and 4 only

16 | P a g e W W W . E D U T A P . C O . I N QUERY? HELLO@EDUTAP.CO.IN / 8146207241


(b)2, 3 and 4 only Enjoyment of above rights is subjected to
(c)1 and 4 only 1. Public order
(d)1, 2, 3 and 4 2. Morality and health and
Answer: C 3. Other provisions relating to fundamental
rights.

QUESTION EXCEPTIONS TO ABOVE RIGHTS


Q. Which Article of the Constitution of India says, The State is permitted to:
“No child below the age of 14 years shall be the 1. Regulate or restrict any economic, financial,
employed to work in any factory or mine or political, or other secular activity associated
engaged in any other hazardous employment”? with religious practice;
(UPSC 2004) 2. Provide for social welfare and reform or
(a) Article 24 throwing open the Hindu religious institutions
(b) Article 45 of a public character to all classes and sections of
(c) Article 330 Hindus.
(d) Article 368 3. Interfere in religious matters for rooting out
certain social evils.
Answer: A - For example, in the past, the Government
has taken steps banning practices like sati,
▪ This Article does not prohibit children’s bigamy or human sacrifice.
employment in harmless or innocent work. IMPORTANT POINT
1. Wearing and carrying of kirpans is to be included
Article 25-28 provides for the Rights to ‘Freedom of in the profession of Sikh religion.
Religion’. 2. This right is available to citizens as well as non -
citizens.
3. Freedom of conscience also includes the right
not to follow any religion.
ARTICLE 25: FREEDOM OF 4. Right to perform tandav dance with lethal
CONSCIENCE AND FREE PROFESSION,
weapons and human skulls in a public procession
PRACTICE AND PROPAGATION OF
was held not to be an essential religious practice
RELIGION
and thus can be banned.
This Article guarantees to every person in India:
ARTICLE 26: FREEDOM TO MANAGE
FREEDOM OF CONSCIENCE RELIGIOUS AFFAIRS
▪ Right to entertain beliefs and doctrines
concerning one’s relation with his God, which Article 26 protects collective freedom of religion. It
are regarded by him to be conducive for his states that every religious denomination or any
spiritual well-being. section shall have the right:
1. To establish and maintain institutions for
RIGHT TO PROFESS, PRACTICE AND PROPAGATE religious and charitable purposes;
Article 25 also covers religious practices (rituals). 2. To manage its own affairs in matters of
▪ Profess means to declare freely and openly religion;
one’s faith and belief. 3. To own and acquire movable and
▪ Practise implies performing the prescribed immovable property; and
religious duties, rites and rituals and exhibiting 4. To administer such property in accordance
his religious beliefs. with law.
▪ Propagate means to communicate the person’s ▪ Each religious denomination or organisation
beliefs to another person. enjoys autonomy in determining what rites and
QUICK FACT ceremonies are essential.
1. Right to propagate does not include the right to convert
another person to one’s own religion.

GROUNDS OF REASONABLE RESTRICTIONS


17 | P a g e W W W . E D U T A P . C O . I N QUERY? HELLO@EDUTAP.CO.IN / 8146207241
It provides for the following rights:
QUICK FACT
1. Any section of the citizens residing in the
A religious denomination is one which satisfies following
three conditions: territory of India or any part having a distinct
(a) It should be a collection of individuals who have a language, script, or culture of its own shall have
system of beliefs (doctrines); the right to conserve the same.
(b) It should have a common organisation; and 2. No citizen shall be denied admission into any
(c) It should be designated by a distinctive name.
educational institution maintained by the State
GROUNDS OF REASONABLE RESTRICTIONS or receiving aid out of State funds on grounds
These rights are subjected to Public order, Morality, only of religion, race, caste, language, or any of
and health. them.
IMPLICATIONS OF ABOVE RIGHTS
ARTICLE 27: FREEDOM FROM
TAXATION FOR PROMOTION OF A ▪ Article 29 grants protection to both religious
RELIGION minorities as well as linguistic minorities.
▪ This Article is not restricted to minorities only as
Article 27 lays down that no person shall be the use of words ‘section of citizens’ includes
compelled to pay any taxes for the promotion or both minorities as well as majority.
maintenance of any particular religion or religious ▪ Right to conserve the language includes the right
denomination. to agitate for the protection of the language.
▪ It prohibits the State from favouring,
patronising, and supporting one particular ARTICLE 30: RIGHT OF MINORITIES TO
religion. ESTABLISH AND ADMINISTER
EDUCATIONAL INSTITUTIONS
IMPLICATION OF ABOVE RIGHT
It implies that: Article 30 grants the following rights to minorities,
1. The taxes can be used for the promotion or whether religious or linguistic:
maintenance of all religions. 1. Right to establish and administer educational
- This is in tune with the concept of secularism institutions of their choice.
as equal respect for all religions. 2. In granting aid, the State shall not discriminate
2. It prohibits only levy of a tax and not a fee. against any educational institution managed by
Therefore, fee can be levied on pilgrims to a minority.
provide them some essential service. 3. Right to impart education to their children in
their own language.
ARTICLE 28: FREEDOM FROM However, the term ‘minority’ has not been defined
ATTENDING RELIGIOUS INSTRUCTION anywhere in the Constitution.
Under Article 28: MODIFICATION BY 44 TH AMENDMENT ACT OF 1978
1. No religious instruction shall be provided in any The compensation amount fixed by the State for the
educational institution wholly maintained out of compulsory acquisition of any property of a minority
State funds. educational institution shall not restrict or abrogate
the rights guaranteed to them.
EXCEPTION
This provision shall not apply to an educational EXCEPTION TO ABOVE RIGHTS
institution administered by the State but ▪ The right to establish and administer educational
established under any endowment or trust. institutions is not absolute.
▪ It does not include the right to maladminister.
2. No person attending any educational institution ▪ State can impose conditions to ensure proper
recognised by the State or receiving aid out of utilization of the aid granted to them.
State funds shall be required to attend any ▪ The Freedom to appoint teachers/lecturers is
religious instruction or worship in that institution subjected to the eligibility conditions prescribed
without his consent. by the State.

ARTICLE 29: PROTECTION OF


INTERESTS OF MINORITIES
18 | P a g e W W W . E D U T A P . C O . I N QUERY? HELLO@EDUTAP.CO.IN / 8146207241
4. The right guaranteed by this article shall not
QUESTION
be suspended except as otherwise provided
Q. The Constitution of India recognises
for by this Constitution.
(UPSC 1999)
- Under Article 359, the President during
(a) Only religious minorities
national emergency can suspend the right to
(b) Only linguistic minorities
move any court for the enforcement of the
(c) Religious and linguistic minorities
fundamental rights.
(d) Religious, linguistic, and ethnic minorities
▪ The rights are guaranteed by the Constitution
not only against executive action but also
Answer: C
against that of legislature.
ARTICLE 32: RIGHT TO ▪ The jurisdiction of the Supreme Court under
CONSTITUTIONAL REMEDIES Article 32 is original but not exclusive.
▪ It is concurrent with the jurisdiction of the high
Abstract declarations of fundamental rights in the court under Article 226.
Constitution are useless unless there is the means to
make them effective.
IMPORTANT POINTS
▪ Article 32 of the Indian Constitution empowered ▪ Supreme Court cannot refuse relief under Article 32 on
the Supreme Court as the defender and the ground that:
guarantor of the fundamental rights of the 1. The aggrieved person may have his remedy under
citizens. ordinary law or from any other court.
2. The disputed facts have to be investigated or
▪ Thus, the right to get the Fundamental Rights
evidence has to be taken before giving relief to
protected is in itself a fundamental right. petitioner.
▪ The Supreme Court has ruled that Article 32 is a 3. The petitioner has not asked for the appropriate
basic feature of the Constitution. writ applicable in his case.
4. Person himself has not approached the court for
remedy.
QUICK FACT ▪ Writ jurisdiction of SC is different from that of HC:
Dr B.R. Ambedkar citing the Importance of Article 1. SC can issue writs for enforcing fundamental rights
32 said that: only while HC can issue writs for other purposes as
“If I were asked to name any particular article in this well. So, the writ jurisdiction of the Supreme Court,
Constitution as the most important- an article without which in this respect, is narrower than that of high court.
this Constitution would be a nullity— I could not refer to any 2. SC cannot refuse remedy under Article 32, but HC
other article except this one. It is the very soul of the can refuse it under Article 226.
Constitution and the very heart of it and I am glad that the
House has realized its importance.” TYPES AND SCOPES OF WRITS
Different kinds of writs mentioned in Articles 32 and
▪ Article 32 contains following provisions: 226 of the Constitution are:
1. The right to move the Supreme Court by 1. Habeas Corpus: ‘To have the body of’
appropriate proceedings for the
It is an order issued by the Court to a person
enforcement of the rights conferred by this who has detained another person, to produce
Part is guaranteed. the body of the latter before it.
- Supreme Court cannot refuse relief for
violation of a substantive fundamental right. ▪ The Court can thus check the legality of the
2. The Supreme Court shall have the power to detention for taking appropriate action.
issue directions or orders or writs for the ▪ The writ can be issued against both public
enforcement of any of the rights conferred authorities as well as private individuals.
by this Part. ▪ This writ is not issued where the:
- Only the rights under Part III are guaranteed 1. Detention is lawful
and can be enforced under Article 32. 2. The proceeding is for contempt of a
3. Parliament may by law empower any other legislature or a court
court to exercise powers to issue directions, 3. Detention is by a competent court
orders, and writs of all kinds. 4. Detention is outside the jurisdiction of the
court.
19 | P a g e W W W . E D U T A P . C O . I N QUERY? HELLO@EDUTAP.CO.IN / 8146207241
3. Administrative authorities.
2. Mandamus: ‘We Command’
5. Quo Warranto: ‘by what authority’
This writ is issued by court to a public official, It is issued by the court to enquire into the
public body, corporation, inferior court, tribunal legality of claim of a person to a public office. It
or Government asking them to perform their cannot be issued in cases of ministerial office or
duties which they have failed or refused to private office.
perform.
▪ The conditions necessary for issue of this writ
▪ This writ is not issued: are:
1. Against a private individual or body 1. The office should be a public office.
2. To enforce departmental instruction that 2. It should be created by a statute or by the
does not possess statutory force Constitution itself.
3. When the duty is discretionary and not 3. The office must be a substantive one.
mandatory ▪ The basis of this writ is that the public has an
4. To enforce a contractual obligation interest to see that an unlawful claimant do not
5. Against the President of India or the state usurp a public office.
Governors - When challenged, the claimant is required to
6. Against the Chief justice of a high court present is claim to that post.
acting in judicial capacity. ▪ Unlike the other four writs, this can be sought by
any interested person and not necessarily by
3. Prohibition: 'To forbid’
the aggrieved person.
It is issued by a higher court to a lower court or
tribunal to prevent the latter from exceeding its ARTICLE 33: ARMED FORCES AND
jurisdiction or usurping a jurisdiction that it does FUNDAMENTAL RIGHTS
not possess.
According to Article 33 the application of
▪ The objective of this writ is to compel courts to fundamental rights to the members of:
keep themselves within the limits of their 1. Armed forces
jurisdiction. 2. Forces charged with the maintenance of public
▪ It thus demands inactivity (not to take case order like Police and Paramilitary
outside jurisdiction) in some cases. 3. Intelligence agencies and analogous services
▪ The writ of prohibition can be issued only against can be restricted or abrogated by the Parliament.
judicial and quasi-judicial authorities. ▪ The expression ‘members of the armed forces’
▪ It is not available against administrative also covers non-combatants members (like
authorities, legislative bodies, and private barbers, carpenters, mechanics, cooks).
individuals or bodies. ▪ The objective is to ensure proper discharge of
their duties and maintenance of discipline
4. Certiorari: 'To be certified’ amongst them.
▪ A Parliamentary law thus enacted can exclude
It is issued by a higher court to a lower court or
tribunal either to transfer a case pending with the Court Martials from the writ jurisdiction of
the latter to itself or to squash the order of the the Supreme Court and the high courts.
latter in a case.
ARTICLE 34: RESTRICTION ON RIGHTS
▪ It is issued on the grounds of excess of WHEN MARTIAL LAW IS IN FORCE IN
jurisdiction or lack of jurisdiction or error of law. ANY AREA
▪ Thus, while prohibition is available during
▪ It imposes restrictions on fundamental rights
pendency of the proceedings and before the
while martial law is in force in any area within
order is made, certiorari can be issued even
the territory of India.
after the order is made.
▪ It can be issued only against:
1. Judicial bodies
2. Quasi-Judicial Authorities
20 | P a g e W W W . E D U T A P . C O . I N QUERY? HELLO@EDUTAP.CO.IN / 8146207241
▪ Therefore the 44th Amendment Act of 1978
QUICK FACT
abolished the right to property as a fundamental
▪ The concept of martial law has been borrowed in India
from the British law. Martial law has not been defined right by repealing Article 19(1) (f) and Article 31
anywhere in the Constitution. from Part III.
▪ In simple word it means ‘rule by military’. ▪ The Act inserted a new Article 300A in Part XII as
▪ It is imposed under the extraordinary circumstances like the ‘Right to Property’ making it a Legal and
war, invasion, insurrection, rebellion, riot, or any Constitutional right.
violent resistance to law to repel force by force for
maintaining or restoring order in the society. GENERAL LIMITATIONS TO FUNDAMENTAL
RIGHTS
▪ It empowers the Parliament to indemnify any The scope of operation of fundamental rights is
Government servant or others for any act done limited in following three cases:
by him during the maintenance or restoration of
order in any area where martial law was in force. CASE 1
▪ The Act of Indemnity made by the Parliament Article 31A, which is, saving of laws providing for
cannot be challenged in any court on the ground acquisition of estates, etc. on the ground of
of contravention of any of the fundamental contravention of the Article 14 and Article 19.
rights. ▪ It saves following five categories of law from
▪ Only the writ of habeas corpus is not suspended being challenged:
during martial rule. (a) Acquisition of estates and related rights by
the State;
ARTICLE 35: LEGISLATION TO GIVE (b) Taking over the management of properties
EFFECT TO THE PROVISIONS OF THIS by the State;
PART (c) Amalgamation of corporations;
(d) Extinguishment or modification of rights of
According to this Article: directors or shareholders of corporations;
1. Parliament shall have, and the legislature of a and
State shall not have, power to make laws: (e) Extinguishment or modification of mining
(a) With respect to any of the matters under leases.
Article 16, Article 32, Article 33, and Article ▪ This Article also provides for the payment of
34. compensation at market value when a person’s
(b) Prescribing punishment for those acts which cultivable land is acquired by state and the land
are declared to be offences under this Part. It is within the statutory ceiling limit.
includes:
- Article 17 related to untouchability CASE 2
- Article 23 related to traffic in human beings Article 31B, which, saves certain Acts and
and forced labour Regulations included in the 9th Schedule on the
2. Any law in force at the commencement of the ground of contravention of any of the fundamental
Constitution related to any of the above matters rights.
will continue in force until modified by the
Parliament. NINTH SCHEDULE
The first Amendment to the Indian Constitution added the
RIGHT TO PROPERTY AT PRESENT Ninth Schedule to it.
Originally, the right to property was one of the seven ▪ It contains a list of central and state laws which cannot
fundamental rights under Part III of the Constitution. be challenged in courts. Currently, 284 such laws are
▪ Since the commencement of the Constitution, shielded from judicial review.
the fundamental right to property has been the ▪ The aim was to provide a protective umbrella to land
most controversial. reforms laws to save them from being challenged in
courts.
- It has caused confrontations between the ▪ In a landmark ruling in IR Coelho vs State of Tamil Nadu,
Supreme Court and the Parliament. SC ruled that the laws placed in the Ninth Schedule are
- It has led to a number of Constitutional open to judicial scrutiny and that such laws do not
amendments. enjoy a blanket protection.
- So, all laws placed under Ninth Schedule after April
24, 1973 shall be open to challenge in court if they
21 | P a g e W W W . E D U T A P . C O . I N QUERY? HELLO@EDUTAP.CO.IN / 8146207241
violated fundamentals rights guaranteed under 3. Many words like ‘public order’, ‘minorities’,
Articles 14, 15, 19 and 21 or the ‘basic structure’ of ‘public interest’ and so on are not clearly
the Constitution.
defined.
4. Fundamental rights are not sacrosanct or
immutable.
- Parliament can curtail or abolish them.
5. It is alleged that the Constitution was
made by the lawyers for the lawyers.
CASE 3 6. These rights can be suspended during National
Article 31C, which provides for, saving of laws giving Emergency.
effect to certain directive principles in Article 39 (b) 7. The critics assert that the provision for
or (c). preventive detention takes away the spirit and
▪ This Article is inserted by the 25th Constitutional substance of the chapter on fundamental rights.
Amendment Act of 1971. It states that:
“No law that seeks to implement the directive
principles specified in Article 39 (b) or (c) shall be void RECENT ISSUES
on the ground of contravention of the fundamental
RESERVATION FOR SC/ST IN PROMOTIONS
rights conferred by Article 14 or Article 19”. In the M. Nagaraj vs Union of India Case (2006), the
RIGHTS OUTSIDE PART III Supreme Court upheld the constitutional validity of
They are: reservations for SCs and STs to include promotions
1. No tax shall be levied or collected except by with three conditions:
authority of law (Article 265 in Part XII). 1. Quantifiable data on the backwardness of SC
2. Freedom of Trade, commerce, and intercourse /STs
throughout the territory of India (Article 301 in 2. Facts about their inadequate representation
Part XIII). 3. Overall administrative efficiency is not
3. Universal Adult suffrage (Article 326 in Part XV). affected.

SIGNIFICANCE OF FUNDAMENTAL ▪ The Centre then approached Supreme Court that


RIGHTS the verdict in the M Nagraj case put unnecessary
conditions in granting quota benefits.
Fundamental Rights are significant because: ▪ Thus, in Jarnail Singh vs Lachhmi Gupta Case
1. They are the bedrock of democratic system, (2018) Supreme Court allowed for grant of quota
secular principle, equality, and social justice. for promotions in the Government jobs to SCs
2. They are necessary for the material and moral and STs without the need to "collect
protection of man. quantifiable data".
3. They facilitate the establishment of rule of law ▪ The court also asked the Government to examine
in the country. the possibility of introducing creamy layer for
4. They protect the interests of minorities and SCs/STs by saying that if some sections bag all the
weaker sections of society. coveted jobs, it will leave the rest of the class as
5. They act as a limit on the authority of the State. backward as they always were.
CRITICISM OF FUNDAMENTAL RIGHTS RESERVATION FOR PERSON WITH DISABILITIES IN
IN INDIAN CONSTITUTION PROMOTION
A three-judge Bench of the Supreme Court has
1. These rights are subjected to innumerable granted relief to persons with disabilities across the
exceptions and restrictions. country by allowing reservation in promotion in all
- Critics remarked that the Constitution grants groups for those employed in public sector jobs.
fundamental rights with one hand and takes ▪ The Bench led by Justice Rohinton F. Nariman
them away with the other. confirmed that 3% reservation should be given
2. They mainly consists of political rights, not social to disabled persons both in direct recruitment
and economic ones. and in promotions.

22 | P a g e W W W . E D U T A P . C O . I N QUERY? HELLO@EDUTAP.CO.IN / 8146207241


▪ The Bench, was answering a reference on the ▪ The order further adds that the state is not
question whether the disabled, included under bound to make a reservation for SCs and STs in
“The Persons with Disabilities (Equal matters of promotions.
Opportunities, Protection of Rights and Full
ACCESS TO INTERNET IS A BASIC RIGHT
Participation) Act, 1995”, were eligible for
Recently, Supreme Court declared that access to the
reservation in promotion.
internet is protected under Article 19 of the
▪ The judgment confirms a 2016 verdict of the top
Constitution.
court in Rajeev Kumar Gupta’s case that granted
▪ In response to a plea against the suspension of
reservation in promotion to disabled persons
internet services in Jammu and Kashmir since last
employed in Groups A and B in addition to
August, a three-judge bench of the Court
Groups C and D categories in the public sector.
affirmed that the
Right to freedom of speech and expression, as
▪ SC held that Indira Sawhney case which capped
guaranteed to all citizens under the first section of
reservation at 50% and prohibited reservation in
that article, covers the right to go online.
promotions pertained only to Backward Classes
(BC) not to PwDs.
▪ The restrictions on internet have to follow the
▪ It held that the ceiling of 50% reservation
principles of proportionality under Article 19(2).
applies only to reservation in favour of BCs
▪ The doctrine essentially signifies that the
under Article 16(4) whereas the reservation in
punishment should not be disproportionate to
favour of PwDs is horizontal, which is under
the offence committed or the nature and extent
Article 16(1).
of the State’s interference with the exercise of a
RESERVATION FOR EWS right must be proportionate to the goal it seeks
The Central Government recently told the Supreme to achieve.
Court that State Governments were free to decide ▪ Freedom of trade and commerce through
whether to implement the 10% reservation for the internet is also a constitutionally protected right
economically backward in jobs and admissions. under Article 19(1)(g).
▪ This reservation of up to 10% for the EWS will be ▪ Suspension of internet for indefinite period not
in addition to the existing reservation cap of permissible.
50% reservation for SC, ST and OBCs.
▪ Centre has no role in deciding the reservation Earlier in the Faheema Shirin vs State of Kerala case
policy of a state government. (2017), Kerala High Court held that the right to have
▪ State governments are free to decide whether to access to the Internet is part of the fundamental
implement the 10% quota for EWS in State right to education as well as the right to privacy
Government jobs and admissions to State under Article 21 of the Constitution.
Government educational institutions.
CONSTITUTIONAL MORALITY
SUPREME COURT SAYS RESERVATIONS FOR JOBS, According to Dr. B. R. Ambedkar:
PROMOTIONS NOT A FUNDAMENTAL RIGHT “Constitutional morality would mean effective
The Supreme Court has held that states are not coordination between conflicting interests of
bound to provide reservation in appointments and different people and the administrative cooperation
there is no fundamental right to claim quota in to resolve them amicably without any confrontation
promotions. amongst the various groups working for the
▪ “No mandamus can be issued by the court realization of their ends at any cost.”
directing the State Government to provide ▪ Constitutional morality is not limited only to
reservations,” the bench said in its verdict. following the constitutional provisions literally
▪ Article 16 (4) and 16 (4A) are in the nature of but is based on values like individual autonomy
enabling provisions, vesting a discretion on the and liberty; equality without discrimination;
State Government to consider providing recognition of identity with dignity; the right to
reservations, if the circumstances so warrant. privacy.
▪ It is settled law that the state cannot be directed
to give reservations for appointment in public DUTIES OF AN INDIAN CITIZEN
posts.
23 | P a g e W W W . E D U T A P . C O . I N QUERY? HELLO@EDUTAP.CO.IN / 8146207241
Original Constitution of India provided only the - In a democratic system, the sovereignty lies
fundamental rights and not the fundamental duties with the people, so defending the
of citizens. However later taking inspiration from sovereignty is also people’s responsibility.
Constitution of erstwhile USSR. (d) to defend the country and render national
▪ A set of ten fundamental duties were service when called upon to do so;
incorporated in the Constitution through 42nd - The National service can took any form. It
Constitutional Amendment Act of 1976. can be fighting on the battlefield or to
▪ 86th Constitutional Amendment Act of 2002 support those fighting.
added one more fundamental duty. - By defending our country, we are defending
our own liberty and freedom.
PRESENT CONSTITUTIONAL POSITION
(e) to promote harmony and the spirit of common
▪ Article 51 A under Part IVA of the Indian
brotherhood amongst all the people of India
Constitution contains 11 fundamental duties.
transcending religious, linguistic, and regional
▪ These duties were included based on the
or sectional diversities; to renounce practices
recommendations of Swaran Singh Committee.
derogatory to the dignity of women;
- It promotes the objective of fraternity in our
Rights are correlative with Duties. It means citizens
preamble.
while enjoying fundamental rights should be
- Harmony is must for India to uphold its
conscious about their duties towards the state and
foundation of “unity with diversity”.
fellow citizens.
- The respect for women come normally to a
LIST OF FUNDAMENTAL DUTIES country where ideals said that gods reside
where women are worshipped.
It shall be the duty of every citizen of India: - Protection of Civil Rights Act (1955) provides
(a) to abide by the Constitution and respect its for punishments for offences related to caste
ideals and institutions, the National Flag, and the and religion.
National Anthem. - Imputation and assertions prejudicial to
- All of us are supposed to maintain the national integration is a punishable offence
dignity of the Constitution by following it in under Section 153 B of IPC.
letter and spirit. - Offences related to religion are also covered
- National flag and National anthem are in Sections 295 – 298 of IPC.
symbol of our history, sovereignty, unity, (f) to value and preserve the rich heritage of our
and pride. composite culture;
- The Prevention of Insults to National - Our cultural heritage is one of the noblest
Honour Act (1971) prevents disrespect to the and richest and acts as great guiding force.
Constitution of India, the National Flag, and - We inherited not only tangible heritage but
the National Anthem. also intangible in the form of rich culture.
(b) to cherish and follow the noble ideals which - Our past has shown us the path of peace,
inspired our national struggle for freedom; love, nonviolence, and truth. It is the
- The battle of freedom was a long one where responsibility of all of us to work for the
thousands sacrificed their lives. preservation of this rich heritage.
- It was not only a political struggle but also a - Article 50 of Indian Constitution also relates
social and economic one. to the above duty.
- Its ideals were those of building a just society (g) to protect and improve the natural environment
and a united nation of freedom, equality, including forests, lakes, rivers, and wildlife, and
nonviolence, brotherhood, & world peace. to have compassion for living creatures;
- We should remain conscious and committed - In light of current menace of increasing
to those ideals to rise above various dividing pollution and environmental degradation, it
tendencies. is our duty to protect and improve our
(c) to uphold and protect the sovereignty, unity, natural environment.
and integrity of India; - It is necessary to balance development with
ecology and to practice sustainable
development.
24 | P a g e W W W . E D U T A P . C O . I N QUERY? HELLO@EDUTAP.CO.IN / 8146207241
- Government actions alone cannot help - There is no provision in Constitution for
bringing about a pollution free clean direct enforcement of duties.
atmosphere. Every citizen should perform 5. The inclusion of fundamental duties has not
his/her duty to bring better results. changed the status of fundamental rights.
- Wildlife (Protection) Act of 1972 prohibits - The purpose of their inclusion is to make
trade in rare and endangered species. citizens aware of their obligations.
- Forest (Conservation) Act of 1980 checks 6. The fulfilment of the fundamental duties are
indiscriminate deforestation and diversion of voluntary and not compulsory.
forest land 7. They serve as a warning against the anti-
(h) to develop the scientific temper, humanism and national and antisocial activities.
the spirit of inquiry and reform; 8. These duties aim to create an enlightened
- Scientific temper and spirit of inquiry citizenship in India, which places public interests
generates a sense of inquisitiveness for above of the personal Interests.
learning from developments around us. 9. The fundamental duties are in consonance with
- Also, Indian Constitution ordains that the Article 29(1) of the Universal Declaration of
Science and technology must be tempered Human Rights, which states that:
with a sense of humanism because “Everyone has duties to the community in which
ultimately the end of all progress is the alone the free and full development of his
human being. personality is possible.”
(i) to safeguard public property and to abjure
violence; CRITICISMS OF FUNDAMENTAL DUTIES
(j) to strive towards excellence in all spheres of
Critics target fundamental duties on following
individual and collective activity so that the
grounds:
nation constantly rises to higher levels of
1. The list does not cover few important duties like
endeavour and achievement;
casting vote, paying taxes.
- Whatever work we take up either as
2. Non-Justiciable nature of fundamental duties.
individual citizens or as groups, our effort
3. Difficulties in understanding few duties and
should be directed towards achieving
phrases like ‘noble ideals’, ‘composite culture’.
excellence.
4. These duties cannot provide any definite
- Excellence means surpassing merit, virtue,
guidance to citizens.
honest performance.
There is need to build adequate awareness and
- Collective activity lays special emphasis on
congenial environment to make citizen proud and
team spirit.
motivated to fulfill his duties.
(k) who is a parent or guardian to provide
opportunities for education to his child or, as QUICK FACT
the case may be, ward between the age of six ▪ The Government of India appointed a committee to
and fourteen years. “operationalise suggestions to teach fundamental
- It was added by the 86th Constitutional duties to the citizens of India” under the chairmanship
of Justice J.S. Verma in 1998, which submitted its report
Amendment Act of 2002. in October 1999.

FEATURES AND SIGNIFICANCE OF


FUNDAMENTAL DUTIES

1. They are the combination of moral and civic


duties.
2. They are confined only to the Indian citizens not
foreigners.
3. They are also non-justiciable in nature. There is
no legal sanction against their violation.
4. Parliament can enforce fundamental duties by
appropriate legislation.

25 | P a g e W W W . E D U T A P . C O . I N QUERY? HELLO@EDUTAP.CO.IN / 8146207241


QUESTION QUESTION
Q. In the context of India, which one of the Q. Under the Constitution of India, which one of
following is the correct relationship between the following is not a fundamental duty?
Rights and Duties? (UPSC 2017) (UPSC 2011)
(a) Rights are correlative with Duties. (a) To vote in public elections
(b) Rights are personal and hence independent of (b) To develop the scientific temper
society and Duties. (c) To safeguard public property
(c) Rights, not Duties, are important for the (d) To abide by the Constitution and respect its
advancement of the personality of the citizen. ideals.
(d) Duties, not Rights, are important for the Answer: A
stability of the State.
Answer: A
QUESTION
Q. To uphold and protect the Sovereignty, Unity
and Integrity of India" is a provision made in the
QUESTION
(UPSC 2015)
Q. Which of the following statements is/are true
(a) Preamble of the Constitution
of the Fundamental Duties of an Indian citizen?
(b) Directive Principles of State Policy
(UPSC 2017)
(c) Fundamental Rights
1. A legislative process has been provided to
(d) Fundamental Duties
enforce these duties.
2. They are correlative to legal duties.
Answer: D
Select the correct answer using the code given
below:
(a) 1 only
(b) 2 only
(c) Both 1 and 2
(d) Neither 1 nor 2
Answer: D

QUESTION
Q. Which of the following is/are among the
Fundamental Duties of citizens laid down in the
Indian Constitution? (UPSC 2012)
1. To preserve the rich heritage of our composite
culture.
2. To protect the weaker sections from social
injustice.
3. To develop the scientific temper and spirit of
inquiry.
4. To strive towards excellence in all spheres of
individual and collective activity.
Select the correct answer using the codes given
below :
(a) 1 and 2 only
(b) 2 only
(c) 1, 3 and 4 only
(d) 1, 2, 3 and 4
Answer: C

26 | P a g e W W W . E D U T A P . C O . I N QUERY? HELLO@EDUTAP.CO.IN / 8146207241


▪ They seek to establish economic and social
democracy in the country.
▪ They aimed to transform India into a ‘welfare
state’.
▪ They envisages an end to economic exploitation
DIRECTIVE PRINCIPLES OF STATE and staggering inequalities and inequities.
POLICY ▪ These principles seeks to fulfill the basic needs
of the common man and to change the structure
Directive Principles of State policy in Part IV of Indian of our society.
Constitution is a unique feature that acts as
Constitutional guidelines to the State. SANCTION BEHIND DIRECTIVE PRINCIPLES
▪ The chapter on the Directive Principles in Indian Though the Directives are not enforceable by the
Constitution lists mainly three things: courts and no court can make the Government to
1. The goals and objectives that we as a society implement them, yet these principles are declared
should adopt; to be “fundamental’.
- Welfare of the people; ▪ The real sanction behind Directive Principles is
- Social, economic, and political justice; political, that is, public opinion.
- Raising the standard of living. ▪ Constitution makers thought that the moral
2. Certain rights that individuals should enjoy apart force behind these guidelines would ensure that
from the Fundamental Rights; the Government would take them seriously.
- Right of adequate livelihood. ▪ They acts as strong weapon in the hand of
- Right to equal pay for equal work for men opposition to discredit the Government on
and women. grounds of not following these principles.
- Right to work.
3. Certain policies that the Government should QUESTION
adopt. Q. The purpose of the inclusion of Directive
- Framing and implementing Uniform civil Principle of State Policy in the Indian Constitution
code. is to establish (UPSC 2002)
- Promotion of cottage industries. (a) Political Democracy
- Promotion of village panchayats. (b) Social Democracy
- Prohibition on intoxicating drugs and liquor. (c) Gandhian Democracy
▪ The Directive Principles are non-justiciable in (d) Social and Economic Democracy
nature, that is, they are not legally enforceable
by the courts for their violation. Answer: D
SOURCE OF DIRECTIVE PRINCIPLES
▪ Constitution of Ireland (1937) was the main QUESTION
source for the Directives in Indian Constitution. Q. ‘Economic Justice’ as one of the objectives of
▪ These Principles resemble the ‘Instrument of the Indian Constitution has been provided in
Instructions’ enumerated in the Government of (UPSC 2013)
India Act of 1935. (a) the Preamble and the Fundamental Rights
- These are the Constitutional instructions or (b) the Preamble and the Directive Principles
recommendations to the State in legislative, (c) the Fundamental Rights and the Directive
executive, and administrative matters. Rights and the Directive Principles of State
AIM OF DIRECTIVE PRINCIPLES Policy
In the words of Justice K. S. Hegde: (d) None of the above
“The purpose of the Directive principles is to fix
certain social and economic goals for immediate Answer: B
attainment by bringing about a non – violent social
revolution.”
▪ They aim at realising the high ideals of justice,
liberty, equality, and fraternity.
27 | P a g e W W W . E D U T A P . C O . I N QUERY? HELLO@EDUTAP.CO.IN / 8146207241
QUESTION QUESTION
Q. The ‘Instrument of Instructions’ contained in Q. The ideal of 'Welfare State' in the Indian
the Government of India Act 1935 have been Constitution is enshrined in its (UPSC 2015)
incorporated in the Constitution of India in the (a) Preamble
year 1950 as (UPSC 2010) (b) Directive Principles of State Policy
(a) Fundamental Rights (c) Fundamental Rights
(b) Directive Principles of State Policy (d) Seventh Schedule
(c) Extent of executive power of State
(d) Conduct of business of the Government of Answer: B
India
▪ The Constitution does not contain any
Answer: B classification of Directive Principles.
▪ However, on the basis of their content and
CONSTITUTIONAL PROVISIONS direction, they can be further classified.
1. On the basis of direction:
Directive Principles of State Policy are enumerated in
Part IV of the Constitution from Articles 36 to 51. • Welfare of people
▪ Article 36 provides for the definition of State in • Social and economic justice
this part. It is same as in Part III. Thus, it includes
1. The legislative and executive organs of the
Goals • Raising Standard of Living
• Promoting International
Central and State Governments, peace
2. All local authorities, and
3. All other public authorities in the country.
• Uniform civil code
▪ Article 37 talks about the application of the
• Promotion of Cottage
principles mentioned in this part. It states that:
1. The provisions contained in this Part shall not
Policies Industries
• Promotion of Village
be enforceable by any court. Panchayats
- So, the directives did not give rise to any
legal right for the violation of which any
individual could seek a remedy. • Adequate Livelihood
2. These principles are fundamental in the • Right against Economic
governance of the country. Rights exploitation
3. It shall be the duty of the State to apply • Right to work
these principles in making laws. • Gender parity in pay
- However, they are not a source of power.
2. On the basis of Content and intent :
QUESTION A. SOCIALISTIC PRINCIPLES
Q. According to the Constitution of India, which These principles reflect the ideology of democratic
of the following are fundamental for the socialism and aims to establish democratic socialist
governance of the country? (UPSC 2013) state of India.
(a) Fundamental Rights
(b) Fundamental Duties
(c) Directive Principles of State Policy DEMOCRATIC SOCIALISM
Democratic socialism is a political philosophy supporting
(d) Fundamental Rights and Fundamental Duties
political democracy within a socially owned economy, with
a particular emphasis on economic democracy.
Answer: C
They includes:
1. Article 38- State shall strive:
(a) To promote the welfare of the people by
securing a social order permeated by
Justice—social, economic, and political
28 | P a g e W W W . E D U T A P . C O . I N QUERY? HELLO@EDUTAP.CO.IN / 8146207241
(b) To minimise inequalities in income, status, 8. First part of Article 47- It direct state to raise the
facilities, and opportunities. level of nutrition and the standard of living of
- This second provision was added by the 44th people and to improve public health.
Constitutional Amendment Act of 1978.
QUESTION
2. Article 39- State shall secure through its policies:
Q. Consider the following statements
(a) the right to adequate means of livelihood
1. There is no provision in the Constitution of
for all citizens;
India to encourage equal pay for equal work
(b) the equitable distribution of material
for both men and women.
resources of the community for the common
2. The Constitution of India does not define
good;
backward classes.
(c) prevention of concentration of wealth and
Which of the statement(s) given above is/are
means of production in the economic
correct? (UPSC 2006)
system;
(a) Only 1
(d) equal pay for equal work for men and
(b) Only 2
women (not a fundamental right but a
(c) Both 1 and 2
Constitutional goal);
(d) Neither 1 nor 2
(e) preservation of the health and strength of
workers and children against forcible abuse;
Answer: B
- Citizens ae not forced by economic necessity
to enter jobs unsuited to their age or B. GANDHIAN PRINCIPLES
strength. They are inspired from the elements of Constructive
(f) Opportunities for healthy development of programme started by Gandhi during the national
children. movement. It contains:
- The last point (f) was modified by the 42nd 1. Article 40- The State shall take steps:
Constitutional Amendment Act of 1976. (a) To organize village panchayats.
(b) Endow them with such powers and authority
3. Article 39 A- The purpose of this article is to which is necessary for their functioning as
promote equal justice and to provide free legal units of self-Government.
aid to the poor.
- This Directive was added by the 42nd 2. Second part of Article 43- State shall promote
Constitutional Amendment Act of 1976. cottage industries on an individual or co-
operation basis in rural areas.
4. Article 41- The State shall secure the right to
work, to education and to public assistance in 3. Article 43 B- the State shall endeavour to
cases of unemployment, old age, sickness, and promote voluntary formation, autonomous
disablement. functioning, democratic control, and
professional management of co-operative
5. Article 42- State shall make provision for societies.
securing just and humane conditions of work - This Directive was added by the 97th
and for maternity relief. Constitutional Amendment Act of 2011.
6. First part of Article 43- State shall make
provision to secure a living wage, a decent 4. Article 46- State shall:
standard of life and social and cultural (a) Promote the educational and economic
opportunities for all workers. interests of SCs, STs, and other weaker
sections of the society.
7. Article 43 A- State shall take steps to secure the (b) Protect them from social injustice and
participation of workers in the management of exploitation.
industries.
- This Directive was added by the 42nd
Constitutional Amendment Act of 1976.

29 | P a g e W W W . E D U T A P . C O . I N QUERY? HELLO@EDUTAP.CO.IN / 8146207241


5. Second part of Article 47- State shall strive to
prohibit the consumption of intoxicating drinks 5. Article 49- It shall be the duty of the State to
and drugs which are injurious to health. protect monuments or places or objects of
artistic or historic interests, declared to be of
6. First part of Article 48- State shall take steps to national importance.
prohibit the slaughter of cows, calves and other
milch and draught cattle and to improve their 6. Article 50- State shall take steps to separate the
breeds. judiciary from the executive in the public
services of the State.
7. Article 51- State shall endeavour to:
QUESTION
(a) Promote international peace and security;
Q. Which of the following is/are included in the
(b) Maintain just and honourable relations
Directive Principles of State Policy?
between nations;
1. Prohibition of traffic in human beings and
(c) foster respect for international law and
forced labour.
treaty obligations;
2. Prohibition of consumption except for
(d) encourage settlement of international
medicinal purposes of intoxicating drinks and
disputes by arbitration.
of other drugs which are injurious to health.
Select the correct answer using the code given
below: (UPSC 2008) QUESTION
(a) Only 1 Q. Which of the following Articles of the Directive
(b) Only 2 Principles of State Policy deals with the
(c) Both 1 and 2 promotion of International peace and security?
(d) Neither 1 nor 2 (UPSC 2002; 2014)
(a) Article 51
Answer: B (b) Article 48A
(c) Article 43A
C. LIBERAL-INTELLECTUAL PRINCIPLES (d) Article 41
1. Article 44- State shall endeavour to secure for
the citizens a Uniform Civil Code throughout the Answer: A
territory of India.
QUESTION
2. Article 45- State shall provide early childhood Q. Which principle among the following was
care and education for all children until they added to the Directive Principles of State Policy
complete the age of six years. by the 42nd Amendment to the Constitution?
- This Directive was changed by the 86th (UPSC 2017)
Constitutional Amendment Act of 2002. (a) Equal pay for equal work for both men and
women
(b) Participation of workers in the management of
UNIFORM CIVIL CODE industries
Uniform civil code is an Indian Constitution's mandate to
replace personal laws based on the scriptures and customs
(c) Right to work, education and public assistance
of each major religious community in India with a common (d) Securing living wage and human conditions of
set of rules governing every citizen. work to workers

3. Second part of Article 48- State shall take steps Answer: B


to organise agriculture and animal husbandry
on modern and scientific lines.
QUESTION
Q. Consider the following provisions under the
4. Article 48 A- State shall endeavour to protect
Directive Principles of State Policy as enshrined
and improve the environment and to safeguard
in the Constitution of India
the forests and wildlife of the country.
1. Securing for citizens of India a uniform civil
- This Directive was added by the 42nd
code
Constitutional Amendment Act of 1976.
30 | P a g e W W W . E D U T A P . C O . I N QUERY? HELLO@EDUTAP.CO.IN / 8146207241
2. Organizing village Panchayats
QUESTION
3. Promoting cottage industries in rural areas
Q. Consider the following statements with
4. Securing for all the workers reasonable leisure
reference to the Constitution of India, the
and cultural opportunities
Directive Principles of State Policy constitute
Which of the above are the Gandhian Principles
limitations upon
that are reflected in the Directive Principles of
1. Legislative function
State Policy? (UPSC 2012)
2. Executive function.
(a) 1, 2 and 4 only
Which of the above statements is/are correct?
(b) 2 and 3 only
(a) 1 only (UPSC 2017)
(c) 1, 3 and 4 only
(b) 2 only
(d) 1, 2, 3 and 4
(c) Both 1 and 2
(d) Neither 1 nor 2
Answer: B
Answer: D
CRITICAL EVALUATION OF DIRECTIVE CRITICISM
PRINCIPLES The Directive Principles of State Policy have been
SIGNIFICANCE criticised on the following grounds:
1. They remind of the basic principles of the social 1. Their non-justiciable character reduce their
and economic order, which the Constitution significance vis- a – vis fundamental rights.
aims at building. 2. Arrangement in illogical manner;
2. They have served as useful beacon-lights to the - Directives are not arranged in a logical
courts. manner based on a consistent philosophy.
- Though the courts cannot declare a law to be - They mixes up relatively unimportant issues
invalid on the grounds of violating the with the most vital economic and social
directives, yet the court can uphold the questions.
validity of such law if it seeks to implement a 3. Conservative nature;
directive. - They failed to target the requirements of
3. They amplify the objectives of the Preamble by modern progressive society of 21st Century.
ensuring justice, equality, and liberty. 4. Lack of clarity about their fulfilment, as it
4. They facilitate stability and continuity in depends on the will of the Government and the
domestic and foreign policies. availability of the resources.
5. They complement the Part III by providing for
FUNDAMENTAL RIGHTS VS. DIRECTIVE
social and economic rights.
PRINCIPLES
- Part III provided political rights.
6. They provide an opportunity to people and Distinction between them are;
opposition to control the Government.
- In election people can test the policies and Fundamental Rights Directive Principles
decisions of ruling party. 1. These are negative as 1. These are positive as
they prohibit the State they require the State
from doing certain to do certain things.
things.
2. These are justiciable.
2. These are non-
justiciable.
3. They aim at 3. They aim at
establishing political establishing social and
democracy. economic democracy
4. They have legal 4. These have both
sanctions. moral and political
sanctions.

31 | P a g e W W W . E D U T A P . C O . I N QUERY? HELLO@EDUTAP.CO.IN / 8146207241


5. They are personal 5. They are societarian Parliament made the 1st Amendment Act (1951), the
and individualistic. and socialistic. 4th Amendment Act (1955) and the 17th Amendment
6. The courts are bound 6. The courts cannot Act (1964) to implement some of the Directives.
to declare a law declare a law violative
GOLAKNATH CASE, 1967
violative of any of the of any of the Directive
The Supreme Court ruled that the Parliament cannot
Fundamental Rights as Principles as
take away or abridge any of the Fundamental
unconstitutional unconstitutional and
Rights, which are ‘sacrosanct’ in nature.
invalid.
GOVERNMENT RESPONSE
▪ The Fundamental Rights are the rights of the Parliament reacted by enacting the 24th Amendment
individual citizens guaranteed by the Act (1971) and the 25th Amendment Act (1971).
Constitution. ▪ The first one declared that the Parliament has
▪ The directive principles states various tenets of the power to abridge or take away any of the
a welfare state. Fundamental Rights by enacting Constitutional
▪ The conflict arises when the State needs to Amendment Acts.
implement a directive principle and it infringes ▪ The second one inserted a new Article 31 – C
on the fundamental rights of the citizens. which states that:
- This problem arose when the Government “No law that seeks to implement the directive
sought to pass laws to abolish zamindari principles specified in Article 39 (b) or (c) shall be void
system. on the ground of contravention of the fundamental
- These measures were opposed on the rights conferred by Article 14 or Article 19”.
ground that they violated right to property.
- However, keeping in mind the societal needs This conflict was finally resolved by Judiciary in the
that are greater than the individual Minerva Mills case (1980). The Supreme Court held
interests, the Government amended the that:
Constitution to give effect to the Directive “The Indian Constitution is founded on the bedrock
Principles of State Policy. of the balance between the Fundamental Rights
- This led to a long legal battle. The executive and the Directive Principles. Harmony and balance
and the judiciary took different positions. between the two is an essential feature of the basic
structure of the Constitution.”
GOVERNMENT’S VIEW
The Government claimed that rights can be CERTAIN DIRECTIVE OUTSIDE PART IV
abridged for giving effect to Directive Principles. OF THE CONSTITUTION
- This argument assumed that some rights
were a hindrance to welfare of the people. 1. Article 335 - Claims of SCs and STs to Services
and posts in connection with the affairs of the
JUDICIARY’S VIEW
Union or a State.
The court held the view that fundamental rights
2. Article 350 A - State to provide facilities for
were so important and sacred that they cannot
instruction in the mother-tongue at the primary
be limited even for purposes of implementing
stage of education to children of linguistic
Directive Principles.
minority groups.
IMPORTANT CASES 3. Article 351 - Indian State to promote the spread
of the Hindi language and to develop it.
CHAMPAKAM DORAIRAJAN CASE, 1951
In this case, the Supreme Court ruled that the:
1. In case of any conflict between the Fundamental
Rights and the Directive Principles, former
should prevail.
2. Fundamental Rights could be amended by the
Parliament by enacting Constitutional
Amendment Acts.
GOVERNMENT RESPONSE

32 | P a g e W W W . E D U T A P . C O . I N QUERY? HELLO@EDUTAP.CO.IN / 8146207241


(a) The right to adequate means of livelihood for all
citizens.
(b) The equitable distribution of material resources
of the community for the common good.
(c) Separation of the judiciary from the executive in
the public services of the State.
(d) The promotion of voluntary formation,
autonomous functioning, democratic control
and professional management of Co-operative
societies.

Q4. Consider the following statements:


1. Right to obtain recognition of the association is
not a fundamental right.
2. Freedom of speech and recognition includes the
right to strike as well.
Which of the statements given above is/are correct
in the context of Article 19 of Indian Constitution?
(a) 1 only
(b) 2 only
(c) Both 1 and 2
(d) Neither 1 nor 2
MCQS FOR PRACTICE
Q5. With reference to Fundamental Duties of
Q1. Which of the following statements is/are Citizens of India, which of the following statement
correct? is not correct?
1. Fundamental rights in Indian Constitution are (a) Article 51 A under Part IV of the Indian
inspired from the Bill of Rights of the American Constitution contains 11 fundamental duties.
Constitution. (b) They are non-justiciable in nature.
2. Part III of the Indian Constitution contains a (c) These duties were included based on the
detailed chapter on Directive Principles. recommendations of Swaran Singh Committee.
Select the correct answer using the codes given (d) They are confined only to the Indian citizens not
below: the foreigners.
(a) 1 only
(b) 2 only Q6. Consider the following statements in the
(c) Both 1 and 2 context of the Fundamental Rights in the
(d) Neither 1 nor 2 Constitution of India:
1. Fundamental rights are not absolute in nature.
Q2. Which among the following provision(s) 2. They are defended and guaranteed by the
was/were added to the Directive Principles of State Parliament.
Policy by 42nd Amendment to the Constitution? 3. They promote the ideal of political democracy.
(a) Secure the participation of workers in the Which of the statements given above is/are
management of industries. correct?
(b) Promote equal justice and to provide free legal (a) 1 and 2 only
aid to the poor. (b) 1 and 3 only
(c) Opportunities for healthy development of (c) 1 only
children. (d) None of the above
(d) All of the above
Q7. In relation to the various modifications in
Q3. The 97th Constitutional Amendment Act of Directive Principles of State Policy, arrange the
2011 provided for following features in correct chronological order of
their addition in Part IV of Indian Constitution:
33 | P a g e W W W . E D U T A P . C O . I N QUERY? HELLO@EDUTAP.CO.IN / 8146207241
1. Opportunities for healthy development of
children. Q11. Which of the following statements is/are
2. To provide early childhood care and education correct?
for all children until they complete the age of six 1. Article 21 states that no person shall be deprived
years. of his life or personal liberty except according to
3. To minimise inequalities in income, status, due process of law.
facilities and opportunities. 2. The protection under Article 21 is available only
Select the correct answer using the codes given against the arbitrary executive action.
below: 3. Right to privacy is a fundamental Right under
(a) 2-3-1 Article 21 of the Indian Constitution.
(b) 1-3-2 Select the correct answer using the codes given
(c) 3-1-2 below:
(d) 3-2-1 (a) 1 and 3 only
(b) 3 only
Q8. Which of the following statements is/are not (c) 1 and 2 only
correct? (d) All of the above
1. The concept of ‘equality before law’ is an
element of the concept of ‘Rule of Law’ of Q12. Consider the following provisions of the
American origin. Fundamental Rights in Indian Constitution:
2. It is a positive concept which states that no 1. Prohibition of discrimination on various grounds
person is above the law. (Article 15).
Select the correct answer using the codes given 2. Equality of opportunity in matters of public
below: employment (Article 16).
(a) 1 only 3. Protection in respect of conviction for offences
(b) 2 only (Article 20)
(c) Both 1 and 2 4. Cultural and Educational Rights (Articles 29-30)
(d) Neither 1 nor 2 Which of the above are the rights that are available
to the Indian Citizens only?
Q9. Which among the following is/are not the (a) 1 and 3 only
ground of criticism of the Fundamental Rights in (b) 1,2 and 4 only
the Constitution of India? (c) 1, 2 and 3 only
(a) They consist of political rights, not social and (d) 1, 2, 3 and 4
economic ones.
(b) They are non-justiciable in nature. Q13. Consider the following statements in the
(c) They can be suspended during National context of the differences between the
Emergency. Fundamental Rights and Directive Principles of
(d) None of the above State policy in the Constitution of India:
1. Fundamental rights are positive in character
Q10. Consider the following statements in the whereas Directives are negative.
context of the Habeas Corpus writ of the Supreme 2. Fundamental rights promotes the welfare of the
Court of India: Individual whereas Directives focuses on the
1. It is an order issued by the court to a person who development of the community.
has detained another person, to produce the 3. Fundamental rights are legally enforceable while
body of the latter before it. Directives are not.
2. This writ can be issued against only public Which of the statements given above is/are
authorities. correct?
Which of the statements given above is/are (a) 1 and 2 only
correct? (b) 1 and 3 only
(a) 1 only (c) 2 and 3 only
(b) 2 only (d) All of the above
(c) Both 1 and 2
(d) Neither 1 nor 2
34 | P a g e W W W . E D U T A P . C O . I N QUERY? HELLO@EDUTAP.CO.IN / 8146207241
Q14. In the Part III of the Indian Constitution the 1. State shall make provision for securing just and
definition of State does not include: humane conditions of work and for maternity
(a) Government and Parliament of India. relief.
(b) Government and the Legislature of each of the 2. State shall provide early childhood care and
States. education for all children until they complete the
(c) All local authorities like Municipalities and age of fourteen years.
Panchayats. Select the correct answer using the code given
(d) Private body or agency working Independently of below:
the State. (a) Only 1
(b) Only 2
Q15. Consider the following statements: (c) Both 1 and 2
1. Right to propagate one’s religion includes the (d) Neither 1 nor 2
right to convert another person to one’s own
religion. Q19. Which of the following Articles of the Directive
2. Article 29 of Indian Constitution is restricted to Principles of State Policy deals with the Uniform
Indian Citizens only. Civil Code for Citizens of India?
Which of the statements given above is/are not (a) Article 51
correct? (b) Article 48A
(a) 1 only (c) Article 44
(b) 2 only (d) Article 41
(c) Both 1 and 2
(d) Neither 1 nor 2 Q20. The purpose of the inclusion of Fundamental
Rights in the Indian Constitution is to establish
Q16. Consider the following pairs and identify the (a) Political Democracy
ones which are correctly matched: (b) Equality and dignity of all individuals
Writs Meaning (c) Unity of the nation
1. Mandamus We command (d) All of the above
2. Certiorari To be certified
3. Quo- Warranto By what authority Q21. “To promote equal justice and to provide free
Select the correct answer using the codes given legal aid to the poor" is a provision made in the
below: (a) Preamble of the Constitution
(a) 1 and 2 only (b) Directive Principles of State Policy
(b) 2 and 3 only (c) Fundamental Rights
(c) 1 and 3 only (d) Fundamental Duties
(d) 1, 2 and 3
Q22. Consider the following statements in the
Q17. Which among the following is not the context of the Article 29 of the Constitution of
fundamental Duty of an Indian Citizen? India:
(a) To uphold and protect the sovereignty, unity and 1. It grants protection to both religious minorities
integrity of India. as well as linguistic minorities.
(b) To safeguard public property and to abjure 2. This Article is restricted to minorities only.
violence. Select the correct answer using the codes given
(c) To protect and improve the natural below:
environment. (a) 1 only
(d) To pay taxes regularly. (b) 2 only
(c) Both 1 and 2
(d) Neither 1 nor 2

Q18. Which of the following is/are included in the Q23. With reference to the Directives Principles of
Directive Principles of State Policy? State Policy, which of the following statement is not
correct?

35 | P a g e W W W . E D U T A P . C O . I N QUERY? HELLO@EDUTAP.CO.IN / 8146207241


(a) Article 36-51 under Part IV of the Indian (d) None of the above
Constitution contains these Directive Principles.
(b) They are non-justiciable in nature. Q28. Which one of the following has the power to
(c) These principles are socialistic in nature only. make laws, to give effect to certain specified
(d) They serve as useful beacon-light to the courts. fundamental rights?
(a) The Parliament of India
Q24. Consider the following statements: (b) The Supreme Court of India
1. Article 27 prohibits only levy of a tax and not a (c) The President of India
fee. (d) Parliament as well as State legislatures
2. As per Article 28, no religious instruction shall be
provided in any educational institution wholly Q29. The protection against double jeopardy under
maintained out of State funds. Article 20 of Indian Constitution is available in
Which of the statements given above is/are proceedings before
correct? (a) Courts of law and Judicial tribunal
(a) 1 only (b) Administrative Proceedings
(b) 2 only (c) Departmental Proceedings
(c) Both 1 and 2 (d) All of the above
(d) Neither 1 nor 2
Q30. Which among the following is/are ground(s) of
Q25.Which of the above is/are classified under the reasonable restrictions on the Right of freedom of
Right to freedom of Religion (Article 25 -28)? speech and expression under Article 19 (1) (a)?
1. Prohibition of discrimination on grounds of (a) Security of the State
religion. (b) Public order
2. Freedom of conscience and profess of religion. (c) Friendly relations with foreign States
3. Protection of culture of minorities. (d) All of the above
4. Freedom to manage affairs of religious
institution.
Select the answer using the code given below:
(a) 1 and 2 only
(b) 1, 2 and 4 only
(c) 2 and 4 only
(d) 2, 3 and 4 only

Q26. Which one of the following writs is not


suspended even during the martial rule in any area
under Article 34 of Indian Constitution?
(a) Habeas Corpus
(b) Quo-Warranto ANSWER WRITING PRACTICE
(c) Mandamus
(d) Certiorari Q1. Indian Constitution craftly balances the rights
and duties of its citizen. Discuss (150 Words)
Q27. Which of the following are envisaged by the
Right to freedom in the Constitution of India?
1. Protection in respect of conviction for offences Q2. Discuss the significance of freedom of speech
2. Protection of life and personal liberty and expression in Indian democracy. Do you think
3. Right to elementary and secondary education that the ground of restrictions on this right are
4. Protection against arrest and detention in certain exceptionally high.
cases (150 Words)
Select the answer using the codes given below:
(a) 1, 2, 3 and 4 Q3. Directive Principles of State Policy are
(b) 2 only fundamental in the governance of the country and
(c) 1, 2 and 4 only
36 | P a g e W W W . E D U T A P . C O . I N QUERY? HELLO@EDUTAP.CO.IN / 8146207241
difficult to ignore by Government in power. Q2. Which among the following provision(s)
Comment. was/were added to the Directive Principles of State
(250 Words) policy by 42nd Amendment to the Constitution?
(a) Secure the participation of workers in the
Q4. What are the key features of fundamental rights management of industries.
in Indian Constitution? Also critically analyse the (b) Promote equal justice and to provide free legal
rights under Article 14, 19 and 21 of Indian aid to the poor.
Constitution. (c) Opportunities for healthy development of
(250 Words) children.
(d) All of the above
Q5. Discuss the importance of Right to Equality in Answer: D
Indian democratic system. Also examine Article 16 in Explanation: New Directive Principles of State policy
the light of various judicial pronouncements and added by the 42nd Amendment to the Constitution
constitutional amendments. asks State:
(250 Words) 1. To secure opportunities for healthy development
of children (in Article 39).
2. To promote equal justice and to provide free
legal aid to the poor (Article 39 A).
3. To take steps to secure the participation of
workers in the management of industries (Article
43 A).
4. To protect and improve the environment and to
safeguard forests and wild life (Article 48 A).

Q3. The 97th Constitutional Amendment Act of


2011 provided for
(a) The right to adequate means of livelihood for all
citizens.
(b) The equitable distribution of material resources
of the community for the common good.
(c) Separation of the judiciary from the executive in
the public services of the State.
(d) The promotion of voluntary formation,
ANSWERS & EXPLANATION autonomous functioning, democratic control
Q1. Which of the following statements is/are and professional management of Co-operative
correct? societies.
1. Fundamental Rights in Indian Constitution are Answer: D
inspired from the Bill of Rights of the American Explanation: The 97th Amendment Act of 2011
Constitution. added a new Directive Principle relating to Co-
2. Part III of the Indian Constitution contains a operative societies.
detailed chapter on Directive Principles. ▪ It requires the state to promote voluntary
Select the correct answer using the codes given formation, autonomous functioning, democratic
below: control and professional management of Co-
(a) 1 only operative societies.
(b) 2 only
(c) Both 1 and 2 Q4. Consider the following statements:
(d) Neither 1 nor 2 1. Right to obtain recognition of the association is
Answer: A not a fundamental right.
Explanation: Part III of the Indian Constitution 2. Freedom of speech and recognition includes the
contains a detailed chapter on Fundamental Rights right to strike as well.
(not Directive Principles). Which of the statements given above is/are correct
in the context of Article 19 of Indian Constitution?
37 | P a g e W W W . E D U T A P . C O . I N QUERY? HELLO@EDUTAP.CO.IN / 8146207241
(a) 1 only 1. Opportunities for healthy development of
(b) 2 only children.
(c) Both 1 and 2 2. To provide early childhood care and education
(d) Neither 1 nor 2 for all children until they complete the age of six
Answer: A years.
Explanation: Right to Freedom of Association only 3. To minimise inequalities in income, status,
includes the right to start an association and right to facilities and opportunities.
continue with the association as such, not the right Select the correct answer using the codes given
to obtain recognition of the association. below:
▪ Right to strike is not a right in Indian (a) 2-3-1
Constitution. (b) 1-3-2
(c) 3-1-2
Q5. With reference to Fundamental Duties of (d) 3-2-1
Citizens of India, which of the following statement Answer: B
is not correct? Explanation: The Directive related to the
(a) Article 51 A under Part IV of the Indian Opportunities for healthy development of children
Constitution contains 11 fundamental duties. was added by 42nd Amendment to Constitution in
(b) They are also non-justiciable in nature. 1976.
(c) These duties were included based on the ▪ Directive to State for providing early childhood
recommendations of Swaran Singh Committee. care and education for all children until they
(d) They are confined only to the Indian citizens not complete the age of six years was modified by
foreigners. the 86th Constitutional Amendment Act of 2002.
Answer: A ▪ 44th Amendment Act of 1978 added to provision
Explanation: Article 51 A under Part IVA (not Part IV) to minimise inequalities in income, status,
of the Indian Constitution contains 11 fundamental facilities and opportunities.
duties.
Q8. Which of the following statements is/are not
Q6. Consider the following statements in the correct?
context of the Fundamental Rights in the 1. The concept of ‘equality before law’ is an
Constitution of India: element of the concept of ‘Rule of Law’ of
1. Fundamental rights are not absolute in nature. American origin.
2. They are defended and guaranteed by the 2. It is a positive concept which states that no
Parliament. person is above the law.
3. They promote the ideal of political democracy. Select the correct answer using the codes given
Which of the statements given above is/are below:
correct? (a) 1 only
(a) 1 and 2 only (b) 2 only
(b) 1 and 3 only (c) Both 1 and 2
(c) 1 only (d) Neither 1 nor 2
(d) None of the above Answer: C
Answer: B Explanation: The concept of ‘equality before law’ is
Explanation: Fundamental Rights are defended and an element of the concept of ‘Rule of Law’ of British
guaranteed by the Supreme Court of India. origin. It a negative concept.
▪ They are not absolute but qualified. The state
can impose reasonable restrictions on them. Q9. Which among the following is/are not the
▪ The Fundamental Rights are meant for ground of criticism of the Fundamental Rights in
promoting the ideal of political democracy. the Constitution of India?
(a) It mainly consists of political rights, not social and
Q7. In relation to the various modifications in economic ones.
Directive Principles of State Policy, arrange the (b) They are non-justiciable in nature.
following features in correct chronological order of (c) They can be suspended during National
their addition in Part IV: Emergency.
38 | P a g e W W W . E D U T A P . C O . I N QUERY? HELLO@EDUTAP.CO.IN / 8146207241
(d) None of the above Q12. Consider the following provisions of the
Answer: B Fundamental Rights in Indian Constitution:
Explanation: Fundamental Rights are Justiciable in 1. Prohibition of discrimination on various grounds
nature. Aggrieved person can approach the courts (Article 15).
for their enforcement. 2. Equality of opportunity in matters of public
employment (Article 16).
Q10. Consider the following statements in the 3. Protection in respect of conviction for offences
context of the Habeas corpus writ of the Supreme (Article 20)
Court of India: 4. Cultural and Educational Rights (Articles 29-30)
1. It is an order issued by the court to a person who Which of the above are the rights that are available
has detained another person, to produce the to the Indian Citizens only?
body of the latter before it. (a) 1 and 3 only
2. This writ can be issued against only public (b) 1,2 and 4 only
authorities. (c) 1, 2 and 3 only
Which of the statements given above is/are (d) 1, 2, 3 and 4
correct? Answer: B
(a) 1 only Explanation: Article 20 grants protection against
(b) 2 only arbitrary and excessive punishment to an accused
(c) Both 1 and 2 person, whether citizen or foreigner or legal person
(d) Neither 1 nor 2 like a company or a corporation.
Answer: A
Explanation: The writ of habeas corpus can be issued Q13. Consider the following statements in the
against both public authorities as well as private context of the differences between the
individuals. Fundamental Rights and Directive Principles of
State policy in the Constitution of India:
Q11. Which of the following statements is/are 1. Fundamental rights are positive in character
correct? whereas Directives are negative.
1. Article 21 states that no person shall be deprived 2. Fundamental rights promotes the welfare of the
of his life or personal liberty except according to Individual whereas Directives focuses on the
due process of law. development of the community.
2. The protection under Article 21 is available only 3. Fundamental rights are legally enforceable while
against arbitrary executive action. Directive are not.
3. Right to privacy is a fundamental Right under Which of the statements given above is/are
Article 21 of Indian Constitution. correct?
Select the correct answer using the codes given (a) 1 and 2 only
below: (b) 1 and 3 only
(a) 1 and 3 only (c) 2 and 3 only
(b) 3 only (d) All of the above
(c) 1 and 2 only Answer: C
(d) All of the above Explanation: Fundamental rights are negative as
Answer: B they prohibit the State from doing certain things.
Explanation: Article 21 declares that no person shall ▪ Directive Principles are positive as they require
be deprived of his life or personal liberty except the State to do certain things.
according to procedure established by law.
▪ The protection under Article 21 is available not Q14. In the Part III of the Indian Constitution the
only against arbitrary executive action but also definition of State does not include:
against arbitrary legislative action. (a) Government and Parliament of India.
▪ The Supreme Court in 2017 held that the Right (b) Government and the Legislature of each of the
to Privacy is a fundamental right for Indian States.
citizens under the Article 21 of the Constitution (c) All local authorities like Municipalities and
of India. Panchayats.

39 | P a g e W W W . E D U T A P . C O . I N QUERY? HELLO@EDUTAP.CO.IN / 8146207241


(d) Private body or agency working independently of Explanation:
the State. ▪ Duty to pay tax is not included in the list of
Answer: D fundamental duties despite being recommended
Explanation: According to the Supreme Court, a by the Swaran Singh Committee.
private body or agency working as an instrument of
the State falls within the meaning of the ‘State’ Q18. Which of the following is/are included in the
under Article 12, not otherwise. Directive Principles of State Policy?
1. State shall make provision for securing just and
Q15. Consider the following statements: humane conditions of work and for maternity
1. Right to propagate one’s religion includes the relief.
right to convert another person to one’s own 2. State shall provide early childhood care and
religion. education for all children until they complete the
2. Article 29 of Indian Constitution is restricted to age of fourteen years.
Indian Citizens only. Select the correct answer using the code given
Which of the statements given above is/are not below:
correct? (a) Only 1
(a) 1 only (b) Only 2
(b) 2 only (c) Both 1 and 2
(c) Both 1 and 2 (d) Neither 1 nor 2
(d) Neither 1 nor 2 Answer: A
Answer: A Explanation:
Explanation: Right to propagate one’s religion does ▪ As per the Article 45 of Indian Constitution, State
not include a right to convert another person to shall provide early childhood care and education
one’s own religion. for all children until they complete the age of six
▪ Article 29 is restricted to Indian Citizen only. years (not fourteen).

Q16. Consider the following pairs and identify the Q19. Which of the following Articles of the Directive
ones which are correctly matched: Principles of State Policy deals with the Uniform
Writs Meaning Civil Code for Citizens of India?
1. Mandamus We command (a) Article 51
2. Certiorari To be certified (b) Article 48A
3. Quo- Warranto By what authority (c) Article 44
Select the correct answer using the codes given (d) Article 41
below: Answer: C
(a) 1 and 2 only Explanation:
(b) 2 and 3 only ▪ According to Article 44 of the Indian
(c) 1 and 3 only Constitution, State shall endeavour to secure for
(d) 1, 2 and 3 the citizens a Uniform Civil Code throughout the
Answer: D territory of India.
Explanation:
▪ All three pairs are correctly matched. Q20. The purpose of the inclusion of Fundamental
Rights in the Indian Constitution is to establish
Q17. Which among the following is not the (a) Political Democracy
fundamental Duty of an Indian Citizen? (b) Equality and dignity of all individuals
(a) To uphold and protect the sovereignty, unity and (c) Unity of the nation
integrity of India. (d) All of the above
(b) To safeguard public property and to abjure Answer: D
violence. Explanation:
(c) To protect and improve the natural ▪ Fundamental Rights are meant for promoting the
environment. ideal of political democracy.
(d) To pay taxes regularly.
Answer: D
40 | P a g e W W W . E D U T A P . C O . I N QUERY? HELLO@EDUTAP.CO.IN / 8146207241
▪ They uphold the equality of all individuals, the 1. Article 27 prohibits only levy of a tax and not a
dignity of the individual, the larger public fee.
interest and unity of the nation. 2. As per Article 28, no religious instruction shall be
▪ They prevent the establishment of an provided in any educational institution wholly
authoritarian and despotic rule in the country. maintained out of State funds.
Which of the statements given above is/are
Q21. “To promote equal justice and to provide free correct?
legal aid to the poor" is a provision made in the (a) 1 only
(a) Preamble of the Constitution (b) 2 only
(b) Directive Principles of State Policy (c) Both 1 and 2
(c) Fundamental Rights (d) Neither 1 nor 2
(d) Fundamental Duties Answer: C
Answer: B Explanation:
Explanation: ▪ Article 27 prohibits only levy of a tax and not a
▪ Article 39 A of the Indian Constitution states that fee. Fee can be levied on pilgrims to provide
it is the duty of state is to promote equal justice them some essential service.
and to provide free legal aid to the poor. ▪ In any educational institution wholly maintained
▪ It is part of the Directive Principles of State out of State funds, no religious instruction shall
Policy. be given.

Q22. Consider the following statements in the Q25.Which of the above is/are classified under the
context of the Article 29 of the Constitution of Right to freedom of Religion (Article 25 -28)?
India: 1. Prohibition of discrimination on grounds of
1. It grants protection to both religious minorities religion.
as well as linguistic minorities. 2. Freedom of conscience and profess of religion.
2. This Article is restricted to minorities only. 3. Protection of culture of minorities.
Select the correct answer using the codes given 4. Freedom to manage affairs of religious
below: institution.
(a) 1 only Select the answer using the code given below:
(b) 2 only (a) 1 and 2 only
(c) Both 1 and 2 (b) 1, 2 and 4 only
(d) Neither 1 nor 2 (c) 2 and 4 only
Answer: A (d) 2, 3 and 4 only
Explanation: Answer: C
▪ Article 29 is not restricted to minorities only as
the use of words ‘section of citizens’ includes Q26. Which one of the following writs is not
both minorities as well as majority. suspended even during the martial rule in any area
under Article 34 of Indian Constitution?
Q23. With reference to the Directives Principles of (a) Habeas Corpus
State Policy, which of the following statement is not (b) Quo-Warranto
correct? (c) Mandamus
(a) Article 36-51 under Part IV of the Indian (d) Certiorari
Constitution contains these Directive Principles. Answer: A
(b) They are non-justiciable in nature. Explanation:
(c) These principles are Socialistic in nature only. ▪ Supreme Court of India states that the writ of
(d) They serve as useful beacon-light to the courts. habeas corpus is not suspended even during the
Answer: C martial rule in any area under Article 34 of Indian
Explanation: Constitution.
▪ Directive Principles are Socialistic, Gandhian a
well as liberal in nature. Q27. Which of the following are envisaged by the
Right to freedom in the Constitution of India?
Q24. Consider the following statements: 1. Protection in respect of conviction for offences
41 | P a g e W W W . E D U T A P . C O . I N QUERY? HELLO@EDUTAP.CO.IN / 8146207241
2. Protection of life and personal liberty Q30. Which among the following is/are ground(s) of
3. Right to elementary and secondary education reasonable restrictions on the Right of freedom of
4. Protection against arrest and detention in certain speech and expression under Article 19 (1) (a)?
cases (a) Security of the State
Select the answer using the codes given below: (b) Public order
(a) 1, 2, 3 and 4 (c) Friendly relations with foreign States
(b) 2 only (d) All of the above
(c) 1, 2 and 4 only Answer: D
(d) None of the above Explanation:
Answer: C The State can impose reasonable restrictions on the
Explanation: exercise of the freedom of speech and expression
Right to Freedom, under Articles 19-22 of the Indian under Article 19 (1) (a) on the grounds of:
Constitution, contains: ▪ Sovereignty and integrity of India
▪ Protection in respect of conviction for offences ▪ Security of the State
(Article 20) ▪ Friendly relations with foreign States
▪ Protection of life and personal liberty (Article ▪ Public order
21) ▪ Decency or morality
▪ Right to elementary education (Article 21A) ▪ Contempt of court
▪ Protection against arrest and detention in ▪ Defamation
certain cases (Article 22) ▪ Incitement to an offence

Q28. Which one of the following has the power to


make laws, to give effect to certain specified
fundamental rights?
(a) The Parliament of India
(b) The Supreme Court of India
(c) The President of India
(d) Parliament as well as State legislatures
Answer: A
Explanation:
▪ Article 35 of Indian Constitution lays down that
the power to make laws, to give effect to certain
specified fundamental rights shall be vested
only in the Parliament of India.

Q29. The protection against double jeopardy under


Article 20 of Indian Constitution is available in
proceedings before
(a) Courts of law and Judicial tribunal
(b) Administrative Proceedings
(c) Departmental Proceedings
(d) All of the above
Answer: A
Explanation:
▪ The protection against double jeopardy is
available only in proceedings before a court of
law or a judicial tribunal.
▪ It is not available in proceedings before
departmental or administrative authorities as
they are not of judicial nature.

42 | P a g e W W W . E D U T A P . C O . I N QUERY? HELLO@EDUTAP.CO.IN / 8146207241

You might also like